Monday, April 17, 2017

CMA USA Part 1 Gleim 2015

http://www.freeaccountingbooks.com/cma-usa-part-1-gleim-2015/


Free Accounting books provides unlimited PDF books, notes & guides of #Financial, #cost and #management #accounting, #ACCA, #CFA, #CIMA, #CPA, #CMA, #FRM and many more .......
#freeaccountingbooks.com

TABLE OF CONTENTS
Detailed Table of Contents
Preface for CMA Part 1 Candidates
Preparing for and Taking the CMA Exam
Study Unit 1. External Financial Statements and Revenue Recognition
Study Unit 2. Measurement, Valuation, and Disclosure: Investments and Short-Term Items
Study Unit 3. Measurement, Valuation, and Disclosure: Long-Term Items
Study Unit 4. Cost Management Concepts
Study Unit 5. Cost Accumulation Systems
Study Unit 6. Cost Allocation Techniques
Study Unit 7. Operational Efficiency and Business Process Performance
Study Unit 8. Analysis and Forecasting Techniques
Study Unit 9. Budgeting - Concepts, Methodologies, and Preparation
Study Unit 10. Cost and Variance Measures
Study Unit 11. Responsibility Accounting and Performance Measures
Study Unit 12. Internal Controls- Risk and Procedures for Control
Study Unit 13. Internal Controls - Internal Auditing and Systems Controls
vi
viii
1
11
47
83
119
145
179
215
247
285
337
369
393
429
Appendix A: Sample Financial Statements
Appendix B: Glossary of Accounting Terms U.S. to British vs. British to U.S.
Appendix C: ICMA Content Specification Outlines and Cross-References . .
Appendix D: ICMA Suggested Reading List
index
Order Form
459
463
465
469
471
479
vi
DETAILED TABLE OF CONTENTS
Study Unit 1. External Financial Statements and Revenue Recognition
1.1. Concepts of Financial Accounting
1.2. Statement of Financial Position {Balance Sheet)
1.3. Income Statement and Statement of Comprehensive Income
1.4. Statement of Changes in Equity and Equity Transactions . . .
1.5. Statement of Cash Flows
1.6. Revenue Recognition -- Revenue Recognition after Delivery
1.7. Revenue Recognition - Long-Term Construction Contracts .
1.8. Essay Questions
11
14
17
21
24
29
32
44
Study Unit 2. Measurement, Valuation, and Disclosure: Investments and Short-Term
items
2.1. Accounts Receivable
2.2. Inventory - Fundamentals
2.3. Inventory -- Cost Flow Methods
2.4. Inventory Measurement in the Financial Statements — Lower of Cost or
Market
2.5. Classification of Investments
2.6. Equity Method
2.7. Business Combinations and Consolidated Financial Statements
2.8. Different Types of Expenses and Liabilities
2.9. Essay Questions
47
51
55
59
60
64
65
67
81
Study Unit 3. Measurement, Valuation, and Disclosure: Long-Term Items
3.1. Property, Plant, and Equipment
3.2. Impairment and Disposal of Long-Lived Assets
3.3. Intangible Assets
3.4. Leases
3.5. Income Taxes
3.6. Accounting for Bonds and Noncurrent Notes Payable
3.7. Accounting for Pensions
3.8. Essay Questions
Study Unit 4. Cost Management Concepts
4.1. Cost Management Terminology
4.2. Cost Behavior and Relevant Range
4.3. Cost Classification
4.4. Costing Techniques
4.5. Essay Questions
Study Unit 5. Cost Accumulation Systems
5.1. Job-Order Costing
5.2. Process Costing
5.3. Activity-Based Costing
5.4. Life-Cycle Costing
5.5. Essay Questions
Study Unit 6. Cost Allocation Techniques
6.1. Absorption and Variable Costing -- Theory
6.2. Absorption and Variable Costing - Calculations
6.3. Joint Product and By-Product Costing
6.4. Overhead Allocation and Normal Costing -- Theory
6.5. Overhead Allocation and Normal Costing - Calculations
6.6. Allocating Service Department Costs -Theory
6.7. Allocating Service Department Costs- Calculations
6.8. Essay Questions
Study Unit 7. Operational Efficiency and Business Process Performance
7.1. Just-in-Time Inventory and Lean Operation
7.2. Enterprise Resource Planning and Outsourcing
7.3. Theory of Constraints and Throughput Costing
7.4. Capacity Management
7.5. Value-Chain Analysis
7.6. Other Process Improvement Tools
7.7. Essay Questions
83
87
89
92
96
101
105
117
119
122
125
128
141
145
148
155
162
177
179
186
186
190
196
197
200
213
215
218
221
225
227
230
245
Detailed Table of Contents vii
;!
Page
Study Unit 8. Analysis and Forecasting Techniques
8.1. Correlation and Regression
8.2. Learning Curve Analysis
8.3. Time Series Analysis
8.4. Expected Value
8.5. Sensitivity Analysis
8.6. Strategic Management
8.7. The Balanced Scorecard
8.8. Strategic Planning
8.9. Essay Question
Study Unit 9. Budgeting - Concepts, Methodologies, and Preparation
9.1. Roles of Budgets and the Budgeting Process
9.2. Budgeting and Standard Costs
9.3. The Master Budget
9.4. Budget Methodologies
9.5. Operating Budget Calculations -- Production and Direct Materials
9.6. Operating Budget Calculations - Others
9.7. Projecting Cash Collections
9.8. The Cash Budget
9.9. Sales Forecasts and Pro Forma Financial Statements
9.10. Essay Questions
Study Unit 10. Cost and Variance Measures
10.1. Variance Analysis Overview
10.2. Static and Flexible Budget Variances
10.3. Direct Materials Variances
10.4. Direct Labor Variances
10.5. Mix and Yield Variances
10.6. t Overhead Variances
10.7. Comprehensive Example
10.8. Sales Variances
10.9. Essay Questions
Study Unit 11. Responsibility Accounting and Performance Measures
11.1. Responsibility Centers
11.2. Performance Measures — Cost, Revenue, and Profit Centers . . .
11.3. Performance Measures — Investment Centers
11.4. Comparing Performance Measures for Investment Centers ....
11.5. Allocating Common Costs
11.6. Transfer Pricing
11.7. Essay Questions
Study Unit 12. Internal Controls -Risk and Procedures for Control
12.1. Risk and the Control Environment
12.2. Control Procedures
12.3. Corporate Governance and Legal Aspects of Internal Control . . .
12.4. Essay Questions
Study Unit 13. Internal Controls -Internal Auditing and Systems Controls
13.1. Internal Auditing
13.2. Systems Controls
13.3. Security Measures
13.4. Essay Questions
247
250
252
254
256
258
265
269
283
285
292
293
296
299
301
305
306
307
332
337
341
344
346
347
349
351
353
367
369
370
372
374
375
377
391
393
402
407
427
429
435
441
457
STUDY UNIT ONE
EXTERNAL FINANCIAL STATEMENTS
AND REVENUE RECOGNITION
(24 pages of outline)
1.1 Concepts of Financial Accounting ................................................................................................ 11
1.2 Statement of Financial Position (Balance Sheet) ........................................................................ 14
1.3 Income Statement and Statement of Comprehensive Income ................................................. 17
1.4 . Statement of Changes in Equity and Equity Transactions ....................................................... 21
1.5 Statement of Cash Flows ............................................................................................... 24
1.6 Revenue Recognition - Revenue Recognition after Delivery .................... ............................ 29
1.7 Revenue Recognition ~ Long-Term Construction Contracts .................... .............................. 32
1.8 Essay Questions ................................................... ......................................................................... 44
This study unit is the firs t of three on external financial reporting decisions. The relative weight
assigned to this major topic in Part 1 of the exam is 15%. The three study units are
Study Unit 1: External Financial Statements and Revenue Recognition
Study Unit 2: Measurement, Valuation, and Disclosure: Investments and Short-Term Items
Study Unit 3: Measurement, Valuation, and Disclosure: Long-Term Items
1.1 CONCEPTS OF FINANCIAL ACCOUNTING
1. The Objective of General-Purpose Financial Reporting
a. The objective of general-purpose financial reporting is to report financial information
that is useful in making decisions about providing resources to the reporting entity.
b. The information reported relates to the entity’s economic resources and claims to them
(financial position) and to changes in those resources and claims.
1) Information about economic resources and claims helps to evaluate liquidity,
solvency, financing needs, and the probability of obtaining financing.
c. Users need to differentiate between changes in economic resources and claims
arising from (1) the entity’s performance (income statement) and (2) other events and
transactions, such as issuing debt and equity (balance sheet). Information about
financial performance is useful for
1) Understanding the return on economic resources, its variability, and its
components;
2) Evaluating management; and
3) Predicting future returns.
d. For general-purpose financial statements to be useful to external parties, they must
be prepared in conformity with accounting principles that are generally
accepted in the United States (GAAP).
NOTE: The CMA exam also tests some knowledge of International Financial Reporting
Standards (IFRS). When international standards diverge significantly from U.S. GAAP, the
differences are highlighted. If there is no specification between GAAP and IFRS, use
GAAP.
e. Financial accounting differs from management accounting.
1) Management accounting assists management decision making, planning, and
control. Management accounting information is therefore primarily directed to
specific internal users, and it ordinarily need not follow GAAP.
SU 1: External Financial Statements and Revenue Recognition
2. Users of Financial Statements
a. Users may directly or indirectly have an economic interest in a specific business.
Users with direct interests usually invest in or manage the business, and users with
indirect interests advise, influence, or represent users with direct interests.
1) Users with direct interests include
a) Investors or potential investors
b) Suppliers and creditors
c) Employees
d) Management
2) Users having indirect interests include
a) Financial advisors and analysts
b) Stock markets or exchanges
c) Regulatory authorities
b. External users use financial statements to determine whether doing business with the
firm will be beneficial.
1) Investors need information to decide whether to increase, decrease, or obtain an
investment in a firm.
2) Creditors need information to determine whether to extend credit and under what
terms.
3) Financial advisors and analysts need financial statements to help investors
evaluate particular investments.
4) Stock exchanges need financial statements to evaluate whether to accept a
firm’s stock for listing or whether to suspend the stock’s trading.
5) Regulatory agencies may need financial statements to evaluate the firm’s
conformity with regulations and to determine price levels in regulated industries.
c. Internal users use financial statements to make decisions affecting the operations of
the business. These users include management, employees, and the board of
directors.
1) Management needs financial statements to assess financial strengths and
deficiencies, to evaluate performance results and past decisions, and to plan for
future financial goats and steps toward accomplishing them.
2) Employees want financial information to negotiate wages and fringe benefits
based on the increased productivity and value they provide to a profitable firm.
3. Features of Financial Statements
a. Financial statements are the primary means of communicating financial information to
external parties. Additional information is provided by financial statement notes,
supplementary information (such as management’s discussion and analysis), and
other disclosures. Information typically disclosed in notes is essential to
understanding the financial statements.
1) The notes are considered part of the basic financial statements. They amplify or
explain information recognized in the statements and are an integral part of
statements prepared in accordance with GAAP.
a) Financial statement notes should not be used to correct improper
presentation.
SU 1: External Financial Statements and Revenue Recognition 13
b. A full set of financial statements includes the following statements:
1) Statement of financial position (also called a balance sheet)
2) Income statement
3) Statement of comprehensive income
4) Statement of changes in equity
5) Statement of cash flows
c. To be useful, information presented in the financial statements must be relevant and
faithfully represented. To enhance the usefulness, the information should be
comparable with similar information for (1) other entities and (2) the same entity for
another period or date. Thus, comparability allows users to understand similarities
and differences.
d. Financial statements are prepared under the going-concern assumption, which
means that the entity is assumed to continue operating indefinitely. As a result,
liquidation values are not important. It is assumed that the entity is not going to be
liquidated in the near future.
4. Financial Statement Relationships
a. Financial statements complement each other. They describe different aspects of the
same transactions, and more than one statement is necessary to provide information
for a specific economic decision.
b. The components (elements) of one statement relate to those of other statements.
Among the relationships are those listed below:
1) Net income or loss from the statement of income is reported and accumulated in
the retained earnings account, a component of the equity section of the
statement of financial position.
2) The components of cash and equivalents from the statement of financial position
are reconciled with the corresponding items in the statement of cash flows.
3) Items of equity from the statement of financial position are reconciled with the
beginning balances on the statement of changes in equity.
4) Ending inventories are reported in current assets on the statement of financial
position and are reflected in the calculation of cost of goods sold on the
statement of income.
5) Amortization and depreciation reported in the statement of income also are
reflected in asset and liability balances in the statement of financial position.
NOTE: See Appendix A for a complete set of financial statements. The
complementary relationships among these statements are lettered.
5. Accrual Basis of Accounting
a. Financial statements are prepared under the accrual basis of accounting. Accrual
accounting records the financial effects of transactions and other events and
circumstances when they occur rather than when their associated cash is paid or
received.
1) Revenues are recognized in the period in which they were earned even if the
cash will be received in a future period.
2) Expenses are recognized in the period in which they were incurred even if the
cash will be paid in a future period.
NOTE: Under the cash basis, revenues are recognized when cash is received and
expenses are recognized when cash is paid. Under GAAP, financial statements
cannot be prepared under the cash basis of accounting.
Stop and review! You have completed the outline for this subunit. Study multiple-choice
questions 1 through 3 beginning on page 34.
14 SU 1: External Financial Statements and Revenue Recognition
1.2 STATEMENT OF FINANCIAL POSITION (BALANCE SHEET)
1. Overview
a. The statement of financial position, also called the balance sheet, reports the amounts
of assets, liabilities, equity, and their relationships at a moment in time, such as at the
end of the fiscal year. It helps users to assess liquidity, financial flexibility, and risk.
b. The basic accounting equation presents a perfect balance between the entity’s
resources and its capita! structure. The entity’s resources consist of the assets the
entity deploys in its attempts to earn a return. The capital structure consists of the
amounts contributed by outsiders (liabilities) and insiders (equity).
Assets = Liabilities + Equity
2. Elements o f Balance Sheet
a.
b.
c.
d.
e.
Assets are resources controlled by the entity as a result of past events. They
represent probable future economic benefits to the entity. Examples include
inventory; accounts receivable; investments; and property, plant, and equipment.
Liabilities are present obligations of the entity arising from past events. Their
settlement is expected to result in an outflow of economic benefits from the entity.
Examples include loans, bonds issued by the entity, and accounts payable.
Equity is the residual interest in the assets of the entity after subtracting all its
liabilities. Examples include a company’s common stock, preferred stock, and
retained earnings. Equity is affected not only by operations but also by transactions
with owners, such as dividends and contributions.
1) Investments by owners are increases in equity of a business entity. They result
from transfers of something of value to increase ownership interests. Assets are
the most commonly transferred item, but services also can be exchanged for
equity interests.
2) Distributions to owners are decreases in equity. They result from transferring
assets, providing services, or incurring liabilities. A distribution to owners
decreases the ownership interest in the company.
Assets and liabilities are separated in the statement of financial position into current
and noncurrent categories.
1) Assets are generally reported in order of liquidity.
Some variation of the following classifications is used by most entities:
Assets
Current assets:
Cash
Certain investments
Accounts and notes receivable
Inventories
Prepaid expenses
Noncurrent assets:
Certain investments and funds
Property, plant, and equipment (PPE)
Intangible assets
Other noncurrent assets
Liabilities
Current liabilities:
Accounts payable
Current notes payable
Current maturities of noncurrent liabilities
Noncurrent liabilities:
Noncurrent notes payable
Bonds payable
Equity
Investments by owners
Retained earnings (income reinvested)
Accumulated other comprehensive income
Noncontrolling interest in a consolidated entity
NOTE: A comprehensive example of statement of financial position can be found in
Appendix A.
3. Current and Noncurrent Assets
a. An asset is classified as current on the statement of financial position if it is expected
to be realized in cash or sold or consumed within the entity’s operating cycle or
1 year, whichever is longer.
SU 1: External Financial Statements and Revenue Recognition 15
b. The foliowing are the major categories of current assets: (1) cash and cash
equivalents; (2) certain individual trading, available-for-sale, and held-to-maturity
securities; (3) receivables; (4) inventories; and (5) prepaid expenses, etc.
c. Noncurrent assets are those not qualifying as current.
d. The following are the major categories of noncurrent assets:
1) Investments and funds include nonoperating items intended to be held beyond
the longer of 1 year or the operating cycle. The following assets are typically
included:
a) Investments in securities made to control or influence another entity and
other noncurrent securities.
b) Certain individual trading, available-for-sale, and held-to-maturity securities
may be noncurrent.
c) Funds restricted as to withdrawal or use for other than current operations,
for example, to (1) retire long-term debt, (2) satisfy pension obligations, or
(3) pay for the acquisition or construction of noncurrent assets.
2) Property, plant, and equipment (PPE) are tangible operating items recorded at
cost and reported net of any accumulated depreciation. They include
a) Land and natural resources subject to depletion, e.g., oil and gas
b) Buildings, equipment, furniture, fixtures, leasehold improvements, land
improvements, assets held under capital leases, noncurrent assets under
construction, and other depreciable assets
3) intangible assets are nonfinancial assets without physical substance. Examples
are patents and goodwill.
4. Current and Noncurrent Liabilities
a. Current liabilities are expected to be settled or liquidated in the ordinary course of
business during the longer of the next year or the operating cycle.
1) Generally speaking, current liabilities are expected to be settled or liquidated
within 1 year from the balance sheet date.
b. The following are the major categories of current liabilities:
1) Trade payables for items entering into the operating cycle, e.g., for materials
and supplies used in producing goods or services for sale.
2) Other payables arising from operations, such as accrued wages, salaries,
rentals, royalties, and taxes.
3) Unearned revenues arising from collections in advance of delivering goods or
performing services, e.g., ticket sales revenue.
4) Other obligations expected to be liquidated in the ordinary course of business
during the longer of the next year or the operating cycle. These include
a) Short-term notes given to acquire capital assets
b) Payments required under sinking-fund provisions
c) Payments on the current portion of serial bonds or other noncurrent debt
d) Long-term obligations that are or will become callable by the creditor
because of the debtor’s violation of a provision of the debt agreement at
the balance sheet date
c. Current liabilities do not include short-term debt if an entity
1) Intends to refinance them on a noncurrent basis and
2) Demonstrates an ability to do so.
a) The ability to refinance may be demonstrated by entering into a
refinancing agreement before the balance sheet is issued.
SU 1: External Financial Statements and Revenue Recognition
d. Noncurrent liabilities are those not qualifying as current. The noncurrent portions of
the following items are reported in this section of the balance sheet:
1) Noncurrent notes and bonds
2) Liabilities under capital leases
3) Most postretirement benefit obligations
4) Deferred tax liabilities arising from interperiod tax allocation
5) Obligations under product or service warranty agreements
6) Advances for noncurrent commitments to provide goods or services
7) Deferred revenue
5. Equity
a. Any recognized transaction that does not have equal and offsetting effects on total
assets and total liabilities changes equity. The following are the major items of equity:
1) Capital contributions by owners (par value of common and preferred stock
issued and additional paid-in capital).
a) Additional paid-in (contributed) capital is the amount received in excess of
par vaiue at the time stock was sold.
2) Retained earnings are the accumulated net income not yet distributed to
owners. Retained earnings can be restricted or unrestricted depending on the
board of directors’ intent.
a) Restriction of retained earnings indicates their unavailability for
disbursement as dividends.
3) Treasury stock is the firm’s own stock that has been repurchased.
a) Treasury stock is reported either at cost (as a deduction from total equity)
or at par (as a direct reduction of the relevant contributed capital account).
b) Treasury stock is reported as a reduction of equity.
4) Accumulated other comprehensive income items not included in net income.
6. Balance Sheet Elements Are Permanent Accounts
a. Assets, liabilities, and equity are recorded in permanent (real) accounts. Their
balances at the end of one accounting period (the balance sheet date) are carried
forward as the beginning balances of the next accounting period.
7. Major Note Disclosures
a. The first footnote accompanying any set of complete financial statements is generally
one describing significant accounting policies, such as the use of estimates and rules
for revenue recognition.
b. Footnote disclosures and schedules specifically related to the balance sheet include
1) Investment securities
2) Property, plant, and equipment holdings
3) Maturity patterns of bond issues
4) Significant uncertainties, such as pending litigation
5) Details of capital stock issues
8. Limitations of Balance Sheet
a. The balance sheet shows a company’s financial position at a single point in time;
accounts may vary significantly a few days before or after the publication of the
balance sheet.
b. Many balance sheet items, such as fixed assets, are valued at historical costs, which
may bear no resemblance to the current value of those items. Even those assets
reported at their current fair values may not always faithfully represent what a
company could sell those items for on an open market.
SU 1: External Financial Statements and Revenue Recognition 17
Stop and review! You have completed the outline fo r this su b u n it Study multiple-choice
questions 4 through 8 beginning on page 35.
1.3 INCOME STATEMENT AND STATEMENT OF COMPREHENSIVE INCOME
1. Income Statement Elements
a. The income statement reports the results of an entity’s operations over a period of
time, such as a year.
The Income Equation
Income (Loss) = Revenues + Gains - Expenses - Losses
b. The following are the elements of income statement:
1) Revenues are inflows or other enhancements of assets or settlements of
liabilities (or both) from delivering or producing goods, providing services, or
other activities that qualify as ongoing major or central operations.
2) Gains are increases in equity (or net assets) other than from revenues or
investments by owners.
3) Expenses are outflows or other usage of assets or incurrences of liabilities (or
both) from delivering or producing goods, providing services, or other activities
that qualify as ongoing major or central operations.
4) Losses are decreases in equity (or net assets) other than from expenses or
distributions to owners.
c. All transactions affecting the net change in equity during the period are included in
income except
1) Transactions with owners
2) Prior-period adjustments (such as error correction or a change in accounting
principle)
3) Items reported initially in other comprehensive income
4) Transfers to and from appropriated retained earnings
d. Revenues, expenses, gains, and losses are recorded in temporary (nominal)
accounts because they record the transactions, events, and other circumstances
during a period of time. These accounts are closed (reduced to zero) at the end of
each accounting period, and their balances are transferred to real accounts.
1) For example, income or loss for the period (a nominal account) is closed to
retained earnings (a real account) at the end of the reporting period.
2. Typical Items of Cost and Expense
a. The expense recognition principles are associating cause and effect, systematic
and rational allocation, and immediate recognition.
1) Matching is essentially synonymous with associating cause and effect. Such a
direct relationship is found when the cost of goods sold is recognized in the
same period as the revenue from the sale of the goods.
b. Cost of Goods Sold
1) For a retailer, cost of goods sold is calculated based on changes in inventory:
Beginning inventory $10,000
Plus: net purchases 14,000
Plus: freight-in 1,000
Goods available for sale $25,000
Minus: ending inventory (5,000)
Cost of goods sold $20,000
18 SU 1: External Financial Statements and Revenue Recognition
2) For a manufacturer, cost of goods sold is calculated as follows:
Beginning raw materials inventory $3,000
Purchases during the period 3,000
Ending raw materials inventory (1,000)
Direct materials used in production $5,000
Direct labor costs 5,000
Manufacturing overhead costs (fixed + variable) 4,000
Total manufacturing costs $14,000
Beginning work-in-process inventory 5,000
Ending work-in-process inventory (4,000)
Cost of goods manufactured $15,000
Beginning finished goods inventory 6,000
Ending finished goods inventory (11,000)
Cost of goods sold $10,000
c. Other Expenses
1) General and administrative expenses are incurred for the benefit of the
enterprise as a whole and are not related wholly to a specific function, e.g.,
selling or manufacturing.
a) They include accounting, legal, and other fees for professional services;
officers’ salaries; insurance; wages of office staff; miscellaneous supplies;
and office occupancy costs.
2) Selling expenses are those incurred in selling or marketing.
a) Examples include sales representatives’ salaries, commissions, and
traveling expense; advertising; sales department salaries and expenses,
including rent; and credit and collection costs.
b) Shipping costs are also often classified as selling costs.
d. Interest expense is recognized based on the passage of time. In the case of bonds,
notes, and capital leases, the effective interest method is used.
3. Income Statement Formats
a. The single-step income statement provides one grouping for revenue items and one
for expense items. The single step is the one subtraction necessary to arrive at net
income.
b. The multiple-step income statement matches operating revenues and expenses in a
section separate from nonoperating items. The most common way to present the
income statement is the condensed format of the multiple-step income statement,
which includes only the section totals.
1) The following is an example of a condensed multiple-step income statement.
■ ■-. V F :: '' EXAMPLE
income Statement
• ; Net sales -X Y $200,000 ; /
Cost of goods sold : (150,000)
Gross profit1 ;; :> $ 50,000
. ' Selling expenses (6,000) .
Administrative expenses (5,000)
Income from operations - $ 39,000
. Other revenues and gains 3,500 • ■
Other expenses and losses (2,500) ■: -' v
Income before taxes $ 40,000 :v ...
j'-: v Income taxes (16,000)
Net income $ 24,000
A more detailed example format can be found in Appendix A. ;
SU 1: External Financial Statements and Revenue Recognition 19
4. Reporting irregular Items
a. When an entity reports a discontinued operation or an extraordinary item, it must
be presented in a separate section after income from continuing operations.
1) Because these items are reported after the presentation of income taxes, they
must be shown net of tax.
2) The term “continuing operations” is used only when a discontinued operation is
reported.
b. Discontinued operations, if reported, may have two components:
1) Income or loss from operations of the component that has been disposed of or is
classified as held for sale from the first day of the reporting period until the date
of disposal (or the end of the reporting period if it is classified as held for sale)
2) Gain or loss on the disposal of this component
c. Extraordinary items are income statement items that meet the following two criteria:
1) Unusual in nature and
2) Infrequent in occurrence in the environment in which the entity operates
a) If an item meets only one of the criteria, it should be presented separately
as a component of income from continuing operations.
IFRS Difference
No items are classified as extraordinary, either on the statement of
comprehensive income or in the notes.
d. The following is an example of a condensed income statement that includes
discontinued operations and extraordinary items:
EXAMPLE
Sales $500,000
.. Cost of goods sold (200,000)
Gross profit $ 300,000
General and administrative expenses • (120,000)
Income before taxes ■ $ 180,000
Income taxes (40,000)
Income from continued operations $ 140,000
Discontinued operations:
: Loss from operations of component X, net of taxes $(60,000)
■ Gain on disposal of component X, net of taxes 10,000
. . : . . . ; . Loss on discontinued operations $ (50,000)
Income before extraordinary items $ 90,000
■ Extraordinary item: : ■
Loss from volcanic eruption, net of taxes , (20,000)
Net income $ 70,000
5. Major Note Disclosures
a. Note disclosures and schedules specifically related to the income statement include
the following:
1) Earnings per share
2) Depreciation schedules
3) Components of income tax expense
4) Components of pension expense
20 SU 1: External Financial Statements and Revenue Recognition
6. Limitations of the Income Statement
a. The income statement does not always show all items of income and expense. Some
of the items are reported on a statement of other comprehensive income and not
included in the calculation of net income.
b. The financial statements report accruai-basis results for the period. The company may
recognize revenue and report net income before any cash was actually received. For
example, the data from the income statement itself is not sufficient enough for
assessing liquidity. This statement must be viewed in conjunction with other financial
statements such as the balance sheet and statement of cash flows.
7. Statement of Comprehensive Income
a. Comprehensive income includes all changes in equity (net assets) of a business
during a period except those from investments by and distributions to owners. It
consists of (1) net income or loss (the bottom line of the income statement) and
(2) other comprehensive income (OCf).
1) Certain income items are excluded from the calculation of net income and
instead are included in comprehensive income. The following are the major
items included in other comprehensive income:
a) The effective portion of a gain or loss on a hedging instrument in a cash
flow hedge
b) Unrealized gains and losses due to changes in the fair value of
available-for-sale securities
c) Translation gains and losses for financial statements of foreign operations
d) Certain amounts associated with accounting for defined benefit
postretirement plans
b. Aff items of comprehensive income are recognized for the period in either
1) One continuous financial statement that has two sections, net income and
OCI, or
2) Two separate but consecutive statements.
a) The first statement (the income statement) presents the components of net
income and total net income.
b) The second statement (the statement of OCI) is presented immediately
after the first. It presents a total of OCI with its components and a total of
comprehensive income.
c. The following is an example of a separate statement of comprehensive income:
Net income V ^TOiOOO;
Other comprehensive income (net of tax):
Loss on defined benefit postretirement plans • $(15,000)
v Gains on foreign currency translation.". 6,000 v ' .
• Gains on remeasuring available-for-sale securities / 4,000
Effective portion of losses on cash flow hedges. (3,000)
; Other comprehensive incomo (loss) (8,000)
Total comprehensive income : . $ 62,000
Stop and review! You have completed the outline for this subunit. Study multiple-choice
questions 9 through 15 beginning on page 37.
SU 1: External Financial Statements and Revenue Recognition 21
1.4 STATEMENT OF CHANGES IN EQUITY AND EQUITY TRANSACTIONS
1. Statement of Changes in Equity
a. A statement of changes in equity presents a reconciliation for the accounting period of
the beginning balance for each component of equity to the ending balance.
b. Each change is disclosed separately in the statement. The following are the common
changes in the equity component balances during the accounting period:
1) Net income (loss) for the period, which increases (decreases) the retained
earnings balance.
2) Distributions to owners (dividends paid), which decreases the retained earnings
balance.
3) Issuance of common stock, which increases the common stock balance. If the
amount paid for the stock is above the par value of stock, the balance of
additional paid-in capital is also increased.
4) Total change in other comprehensive income during the period.
EXAMPLE
Cl Company
Statement of Changes in Equity (in millions)
For the Year Ended December 31 Year .1 :
Tota! Retained. Accumulated Other Common Additional Treasury
Equity Earnings Comprehensive Income Stock Paid-in Capital Stock
Beginning balance $500 : $350 ■ $100 $40 . $ 30 ($20)
.. Net income for the period. 110 - 110.
OCI for the period
Common stock issued .
. 25
90
25
10 . 80
Dividends declared :
Repurchase of common stock:
(60)
(15)
: : (60) : ,:>
■ (15)
Ending balance $650 $400 $125. $50 $110 :: $(35)
A comprehensive example of a statement of changes in equity can be found in Appendix A.
2. Statement of Retained Earnings
a. A statement of retained earnings reconciles the beginning and ending balances of the
account. This statement is reported as part of the statement of changes in equity in a
separate column.
1) The following is a common example of retained earnings reconciliation:
Retained earnings beginning balance
+ Net income (loss) for the period
- Dividends distributed during the period
+ Positive (negative) prior-period adjustments
Retained earnings ending balance_______
2) Prior-period adjustments include the cumulative effect of changes in accounting
principle and corrections of prior-period financial statement errors. These items
require retrospective application, i.e., adjustment of the beginning balance of
retained earnings for the prior period’s cumulative effect on the income
statement.
a) Thus, corrections of prior-period errors and the cumulative effect of
changes in accounting principle must not be included in the calculation of
current-period net income.
SU 1: External Financial Statements and Revenue Recognition
b. Retained earnings are sometimes appropriated (restricted) to a special account to
disclose that earnings retained in the business (not paid out in dividends) are being
used for special purposes. An appropriation must be clearly displayed within equity.
1) Purposes include (a) compliance with a bond indenture (bond contract),
(b) retention of assets for internally financed expansion, (c) anticipation of
losses, or (d) adherence to legal restrictions.
2) The appropriation does not set aside assets. It limits the availability of
dividends. A formal entry (debit retained earnings, credit retained earnings
appropriated) or disclosure in a note may be made.
3) Transfers to and from an appropriation do not affect net income.
3. Common and Preferred Stock
a. The most widely used classes of stock are common and preferred. The following basic
terminology is related to stock.
1) Stock authorized is the maximum amount of stock that a corporation is legally
allowed to issue.
2) Stock issued is the amount of stock authorized that has actually been issued by
the corporation.
3) Stock outstanding is the amount of stock issued that has been purchased and is
held by shareholders.
b. The common shareholders are the owners of the firm. They have voting rights, and
they select the firm’s board of directors and vote on resolutions. Common
shareholders are not entitled to dividends unless so declared by the board of
directors. A firm may choose not to declare any.
1) Common shareholders are entitled to receive liq u id a tin g d is tr ib u tio n s only
after all other claims have been satisfied, including those of preferred
shareholders.
2) Common shareholders ordinarily have preemptive rights. Preemptive rights
give current common shareholders the right to purchase any additional stock
issuances in proportion to their ownership percentages. This way the
preemptive rights safeguard a common shareholder’s proportionate interest in
the firm.
c. Preferred stock has features of debt and equity. It is classified as an equity
instrument and presented in the equity section of the firm’s balance sheet. Preferred
stock has a fixed charge, but payment of dividends is not an obligation. The payment
of dividends is at the firm’s discretion. Preferred shareholders tend not to have voting
rights.
1) Preferred shareholders have the right to receive
a) Dividends at a specified fixed rate before common shareholders may
receive any and
b) Distributions before common shareholders, but after creditors, in the event
of firm bankruptcy (liquidation).
2) The following are common features of preferred stock:
a) Cumulative preferred stock accumulates unpaid dividends (called
dividends in arrears). Dividends in arrears must be paid before any
common dividends can be paid.
b) Holders of convertible preferred stock have the right to convert the stock
into shares of another class (usually common stock) at a predetermined
ratio.
SU 1: External Financial Statements and Revenue Recognition 23
4. Equity Transactions - Issuance of Stock
a. Cash is increased (debited), the appropriate stock account is increased (credited) for
the total par value of stock issued, and additional paid-in capital (paid-in capital in
excess of par) is increased (credited) for the difference.
EXAMPLE
A company issued 50,000 shares of its $1 par-vaiue common stock. The market price of the stock was $17 per share on the
day of issue.
Cash (50,000 shares x $17 market price) $850,000
Common stock (50,000 shares x $1 par value) $ 50,000 ' :
Additional paid-in capital common (difference) . 800,000
The balances of cash, common stock, and additional paid-in capital were increased by $850,000, $50,000, and $800,000,
respectively.
b. Direct costs of issuing stock (underwriting, legal, accounting, tax, registration, etc.)
must not be recognized as an expense. Instead, they reduce the net proceeds
received and additional paid-in capital.
5. Equity Transactions - Cash Dividend
a. On the date of declaration, the board of directors formally approves a dividend.
A declaration of a dividend decreases (debits) the retained earnings account.
b. Al! holders of the stock on the date of record are legally entitled to receive the
dividend.
c. The date of payment is the date on which the dividend is paid.
EXAMPLE
On September 12, a company’s board of directors doclared a $3 per-share dividend to be paid on October 15 to all holders
of common stock. On the date of declaration, 40,000 shares of common stock were outstanding.
September 12 -- Declaration date October 15 - Payment date
Retained earnings (40,000 x $3) $120,000 Dividends payable $120,000
Dividends payable $120,000 Cash $120,000
6. Equity Transactions - Property Dividend
a. When an entity declares a dividend consisting of tangible property,
1) First, the property is remeasured to fair value as of the date of declaration, and
any gain or loss on the remeasurement is recognized in the statement of
income
2) Second, the carrying amount of retained earnings is decreased for the fair value
of the property to be distributed
3) Third, the property is distributed as a dividend
EXAMPLE
On August 1, a company’s board of directors declared a property dividend (land) to be distributed on December 1 to holders
of common stock. On August 1, the carrying amount of the land to be distributed is $50,000 and its fair value is $80,000.
The journal entries to record the declaration and distribution of the property dividend are as follows:
Auaust 1 - Declaration date
Land ($80,000-$50,000)
. \ Gain on land remeasurement
$30,000 Retained earnings $80,000
$30,000 Property dividend payable $80,000
December 1 -- Pavment date
Property dividend payable
... Land
$80,000
$80,000
24 SU 1: External Financial Statements and Revenue Recognition
7. Equity Transactions - Stock Dividend and Stock Split
a. A stock dividend involves no distribution of cash or other property. Stock dividends are
accounted for as a reclassification of different equity accounts, not as liabilities.
1) The recipient does not recognize income. It has the same proportionate interest
in the entity and the same total carrying amount as before the stock dividend.
b. The accounting for stock dividends depends on the percentage of new shares to be
issued.
1) An issuance of shares less than 20% to 25% of the previously outstanding
common shares should be recognized as a stock dividend.
2) An issuance of more than 20% to 25% of the previously outstanding common
shares should be recognized as a stock split in the form of a dividend.
c. In accounting for a stock dividend, the fair value of the additional shares issued is
reclassified from retained earnings to common stock (at par value) and the difference
to additional paid-in capital.
; e x a m p l e - - V ^ - ^' v-'' V;-; ; =
On May 1, a company's board of directors declared and paid a 10% stock dividend on the 45,000 shares of common stock
outstanding ($1 par value). Tho stock was trading for $15 per share at tho declaration date.
- Mav 1 - Declaration and payment date
Retained earnings [(45,000 shares x 10%) * $15 market price]. "... $67,500 ;
Common stock [(45,000 shares * 10%) x $1 par value] $ 4,500
Additional paid-in capital (difference) 63,000
d. For a stock dividend that is accounted for as a stock split in the form of a dividend,
the par vaSue of the additionai shares issued is reclassified from retained earnings to
common stock.
e x a m p l e ;
In the preceding example, assume that a 40% stock split in the form of a dividend was declared.
Mav 1 -- Declaration and pavmont date .
Retained earnings [(45,000 shares x 40%) x $1 par value]
Common stock . - - ■ -
$18,000
$18,000
e. Stock splits are issuances of shares that do not affect any aggregate par value of
shares issued and outstanding or total equity. Stock split reduces the par value of
each stock and increases the number of shares outstanding.
1) No entry is made, and no transfer from retained earnings occurs.
Stop and review! You have completed the outline for this subunit. Study muitiple-choice
questions 16 through 20 beginning on page 39.
1.5 STATEMENT OF CASH FLOWS
1. Overview
a. The primary purpose of the statement of cash flows is to provide relevant information
about the cash receipts and cash payments of an entity during the period. To achieve
this purpose, the statement should provide information about cash inflows and
outflows from the operating, investing, and financing activities of an entity. This is the
accepted order of presentation.
1) The statement of cash flows should help users assess the entity’s ability to
generate positive future net cash flows (liquidity), its ability to meet obligations
(solvency), and its financial flexibility.
SU 1: External Financial Statements and Revenue Recognition 25
b. The statement of cash flows explains the change in cash and cash equivalents during
the period. It reconciles the period’s beginning balance of cash and cash equivalents
with the ending balance.
v;: EXAMPLE
The following is an example of the summarized format of the statement of cash flows (only the. headings). The amounts of
cash and cash equivalents at the beginning and end of the year are taken from the balance sheet.:
Entity A’s Statement of Cash Flows for the Year Ended December 31, Year 1
Net.cash provided by (used in) operating activities
Net cash provided by (used in) investing activities
Net cash provided by (used in) financing activities
Net increase (decrease) in cash and cash equivalents during the year
Cash and cash equivalents at beginning of year (January 1, Year 1)
Cash and cash equivalents at end of year (December 31, Year 1)
A more detailed example format can be found in Appendix A.
2. Operating Activities
a. Operating activities are all transactions and other events that are not financing or
investing activities. Cash flows from operating activities are primarily derived from the
principal revenue-producing activities of the entity. They generally result from
transactions and other events that enter into the determination of net income,
b. The following are examples of cash inflows from operating activities:
1) Cash receipts from the sale of goods and services (including collections of
accounts receivable)
2) Cash receipts from royalties, fees, commissions, and other revenue
3) Cash received in the form of interest or dividends
c. The following are examples of cash outflows from operating activities:
1) Cash payments to suppliers for goods and services
2) Cash payments to employees
3) Cash payments to government for taxes, duties, fines, and other fees or
penalties
4) Payments of interest on debt
d. The two acceptable methods of presentation of cash flows from operating activities are
the direct and the indirect methods.
1) The only difference between these two methods is their presentation of net cash
flows from operating activities. The total cash flows from operating,
investing, and financing activities are the same regardless of which method is
used.
2) The CMA exam requires candidates to know how to prepare the statement of
cash flows using the indirect method.
3. Investing Activities
a. Cash flows from investing activities represent the extent to which expenditures have
been made for resources intended to generate future income and cash flows.
b. The following are examples of cash outflows (and inflows) from investing activities:
1) Cash payments to acquire (cash receipts from sale of) property, plant and
equipment; intangible assets; and other long-lived assets
2) Cash payments to acquire (cash receipts from sale and maturity of) equity and
debt instruments of other entities for investing purposes
3) Cash advances and loans made to other parties (cash receipts from repayment
of advances and loans made to other parties)
$20,000
(5,000)
■ 9,000
$24,000
6,000
$30,000
SU 1: External Financial Statements and Revenue Recognition
4. Financing Activities
a. Cash flows from financing activities generally involve the cash effects of transactions
and other events that relate to the issuance, settlement, or reacquisition of the entity’s
debt and equity instruments.
b. The following are examples of cash inflows from financing activities:
1) Cash proceeds from issuing shares and other equity instruments (obtaining
resources from owners).
2) Cash proceeds from issuing loans, notes, bonds, and other short-term or
long-term borrowings.
c. The following are examples of cash outflows from financing activities:
1) Cash repayments of amounts borrowed
2) Payments of cash dividends
3) Cash payments to acquire or redeem the entity’s own shares
4) Cash payments by a lessee for a reduction of the outstanding liability relating to
a capital lease
5. Noncash Investing and Financing Activities
a. Information about all investing and financing activities that affect recognized assets
or liabilities but not cash flows must be disclosed in the notes, outside the body of
the statement of cash flows.
b. The following are examples of noncash investing and financing activities:
1) Conversion of debt to equity
2) Acquisition of assets either by assuming directly related liabilities or by means of
a capital lease
3) Exchange of a noncash asset or liability for another
6. Indirect Method of Presenting Operating Cash Flows
a. Under the indirect method (also called the reconciliation method), the net cash flow
from operating activities is determined by adjusting the net income of a business for
the effect of the following:
1) Noncash revenue and expenses that were included in net income, such as
depreciation and amortization expenses, impairment losses, undistributed
earnings of equity-method investments, and amortization of discount and
premium on bonds
2) Items included in net income whose cash effects relate to investing or
financing cash flows, such as gains or losses on sales of PPE items (related
to investing activities) and gain or losses on extinguishment of debt (related to
financing activities)
3) All deferrals of past operating cash flows, such as changes during the period in
inventory and deferred income
4) All accruals of expected future operating cash flows, such as changes during
the period in accounts receivable and accounts payable
NOTE: The net income for the period as it reported in the income statement was
calculated using the accrual method of accounting. Therefore, adjustments must be
made to reach the amount of cash flow from operating activities.
b. The reconciliation of net income to net cash flow from operating activities must
disclose all major classes of reconciling items. At a minimum, this disclosure reports
changes in (1) accounts receivable and accounts payable related to operating
activities and (2) inventories.
SU 1: External Financial Statements and Revenue Recognition 21
EXAMPLE
During Year 2, Bishop Corp. had the foilowing transactions: ,
Inventory (cost $9,000) was sold for $14,000, with $13,000 on credit and $1,000 in cash;;
. : Cash collected on credit sales to customers was $12,000. .
: ; ® : Inventory was purchased for $6,500.
© Bishop paid $8,500 to suppliers..
The following is Bishop's income statement for the year ended on December 31, Year 2:
Sales $14,000
Cost of goods sold (9,000)
Net income $ 5,000
The following are Bishop’s balance sheets on December 31, Yoar 1, and December 31, Year 2:
December 31 December 31
Current assets Year 1 Year 2 Current liabilities Year 1 Year 2
Cash $10,000 $14,500 Acc. payable $ 5,000 $ 3,000
Net acc. receivable 6,000 7,000 Equity
Inventory 14,000 11,500 Common stock 21,000 21,000
Retained earnings 4,000 . 9,000
Total assets $30,000 $33,000 .. Total liability and equity $30,000 $33,000
Under the indirect method, the net cash flow from operating activities is determined by adjusting net income for the period.
Bishop’s Statement of Cash Flows for the Year Ended on December 31, Year 2
(Using the indirect method)
Cash flows from operating activities:
Net income
Increase in accounts receivable ($7,000 - $6,000)
Decrease in inventory ($11,500 - $14,000)
Decrease in accounts payable ($3,000 - $5,000)
Total adjustments
Net cash provided by operating activities
Cash at beginning of year
Cash at end of year
Explanation:
(1) The amount of accounts receivable increased during year by $1,000, implying that cash collections were.
less than credit sales. The net income for the period is an accrual.accounting amount. Thus, the.increase.
in accounts receivable during the period must be subtracted from net income to determine the cash flow
from operating activities. •
(2) Inventory decreased by $2,500, implying that purchases were less than the. amount of cost of goods sold.
Thus, it must be added to net income to determine the cash flow from operating.activities.
(3) Accounts payable decreased by $2,000, implying that cash paid to suppliers was greater than purchases.
Thus, it must be subtracted from net income..
$(1,000)
2,500
(2,000)
$ 5,000
(1)
(2)
(3)
(500)
$ 4,500
10,000
$14,500
The following rules will help reconcile the net income to net cash flow from operating activities under the
indirect method:
Increase in current operating liabilities Added to net income
Decrease in current operating assets Added to net income
Increase in current operating assets Subtracted from net income
Decrease in current operating liabilities Subtracted from net income
i l iiUI'l' 'III
28 SU 1: External Financial Statements and Revenue Recognition
: : V ^ EXAMPLE .
During Year 2, Knight Corp. had the following transactions:
: On December 31, Year 2, ^ machine was sold for $47,000 in-cash. The machine was acquired by Knight on ^
January 1, Year 1, for $50,000. Its useful life is 10 years with no salvage value, and it is depreciated using the
■straight-line, method.
V:. ® On December 31, Year 2, the $31,500 due on a loan (principal + accumulated interest) was repaid. The term of
: ; the loan was 1 year from December 31, Year 1. The annual interest rate was 5%.
During Year 2, Knight received $14,000 in cash for services provided to customers.
The following is Knight’s income statement for the year ended on December 31, Year 2;
Revenue . $14,000
Depreciation expense ($50,000 * 10) . (5,000)
Interest expense ($30,000 * 5%) (1,500)
. Gain on machine disposal [$47,000 - ($50,000 - $10,000)] 7,000
NetincJome $14,500
The following are Knight’s balance sheets on December 31, Year 1, and December 31, Year 2:
December 31 . December 31
Current assets Year 1 Year 2 Current liabilities Year 1 : Year 2
Cash .. . . . . $10,000 : $39,500 V" Loan $30,000 . $ 0
Fixed assets Equity
Machine at cost : 50,000 . , 0 Common stock 21,000 .21,000
Accumulated depreciation (5,000) 0 . ■ Retained earnings .. . 4,000 .18,500
Net fixed assets . $45,000 $ 0
Total assets $55,000 . $39,500 . - Total liability and equity . $55,000 $39,500
Under the indirect method, the net cash.flow from operating activities is determined by adjusting net income for the period.
Knight’s Statement of Cash Flows for the Year Ended on December 31, Year 2
.. . (Using the indirect method)
Cash flows from operating activities: ;
, Net income
Depreciation expense
Gain on sale of machine
Total adjustments
Net cash provided by operating activities
. Cash flows from investing activities:
Proceeds from sale of machine
Net cash provided by investing activities
Cash flows from financing activities:
Repayment of loan
Net cash used in financing activities
>- ;■Net increase in cash -
Cash at beginning of year
^ Cash at end of year \
. Explanation ■
(1) Depreciation expense is a noncash expense included in net income, Thus, it must be added to net
income to determine the net cash flow from operating activities.
(2) Gain on sale of machine is an item included in net income. Its cash effect is related to investing activities.
Thus, it must be subtracted from net income to determine the net cash flow from operating activities.
.... $14,500
$ 5,000 . ; (1)
(7,000) (2)
(2,000) ...
$12,500
$47,000 '
47.000
(30,000)
(30,000)
v $29,500
10.000
$ 39,500
SU 1: External Financial Statements and Revenue Recognition 29
The following rules wiil help reconcile net income to net cash flow from operating activities under the indirect
method:
Noncash losses and expenses included in net income Added to net income
Losses and expenses whose cash effects are related
to investing or financing cash flows Added to net income
Noncash gains and revenues included in net income Subtracted from net income
Gains and revenues whose cash effects are related to
investing or financing cash flows Subtracted from net income
7. Direct Method of Presenting Operating Cash Flows
a. Under the direct method, the entity presents major classes of actual gross operating
cash receipts and payments and their sum (net cash flow from operating activities).
At a minimum, the following must be presented:
1) Cash collected from customers
2) Interest and dividends received
3) Other operating cash receipts, if any
4) Cash paid to employees and other suppliers of goods and services
5) Interest paid
6) Income taxes paid
7) Other operating cash payments, if any
b. If the direct method is used, the reconciliation of net income to net cash flow from
operating activities (the operating section of the indirect method format) must be
provided in a separate schedule.
1) Most entities apply the indirect method because the reconciliation must be
prepared regardless of the method chosen.
Stop and review! You have completed the outline for this subunit. Study multiple-choice
questions 21 through 26 beginning on page 40.
1.6 REVENUE RECOGNITION - REVENUE RECOGNITION AFTER DELIVERY
1. Revenue Recognition Principle
a. According to the revenue recognition principle, revenues and gains should be
recognized when (1) realized or realizable and (2) earned.
1) Revenues and gains are realized when goods or services have been exchanged
for cash or claims to cash. Revenues and gains are realizable when goods or
services have been exchanged for assets that are readily convertible into cash
or claims to cash.
2) Revenues are earned when the earning process has been substantially
completed, and the entity is entitled to the resulting benefits or revenues.
a) Thus, revenue on sales can be recognized in the statement of income
even if the cash from sales is not received yet.
30 SU 1: External Financial Statements and Revenue Recognition
IFRS Difference
For a sale of goods, revenue is recognized when five conditions are met.
(1) The entity has transferred the significant risks and rewards of ownership,
(2) the entity has neither continuing managerial involvement to an extent
associated with ownership nor effective control over the goods, (3) the
revenue (measured at the fair value of the consideration received or
receivable) can be reliably measured, (4) it is probable that the economic
benefits will flow to the entity, and (5) transaction costs can be reliably
measured.
For the rendering o f a service, if the outcome can be reliably estimated,
revenue (measured as described above) is recognized based on the stage of
completion (the percentage-of-completion method). The outcome can be
reliably estimated when (1) revenue can be reliably measured, (2) it is
probable that the economic benefits will flow to the entity, (3) the stage of
completion can be reliably measured, and (4) the costs incurred and the
costs to complete can be reliably measured.
2. The Installment Method
a. The installment method is only acceptable when receivables are collectible over an
extended period and no reasonable basis exists for estimating the degree of
collectibility.
b. The installment method recognizes a partial profit on a sale as each installment is
collected.
1) This approach differs from the ordinary procedure, that is, recognition of revenue
when a transaction is complete. Thus, when collection problems (bad debts)
can be reasonably estimated, the full profit is usually recognized in the period of
sale.
c. The amount recognized each period under the installment method is the realized
gross profit. This amount equals the cash collected on installment sales for the
period times the gross profit percentage on installment sales for the period (a
separate gross profit percentage is calculated for each period).
Gross profit on installment sales
Gross profit percentage ~ ---------------------------------------------------------- ------------------------ Installment sales
EXAMPLE;
A TV costing $600 is the only item sold on the installment basis in Yea
November 1, Year 1. Thus, the gross profit percentage is 40% [($1,00(
anrl th^ tc Hi in1 nlrto rniAnfK!v/ r\^x/nrV^ntG: nf $ Inn '<
r 1. The TV was sold for a price of $1,000 on
) - $600) t $1,000]. A down payment of $100 was
sq/'K Roi/'Qi f o i l niQwniDnfG’ J.CVCIVCU, ell lu ll Ic I pl IJaJi lUCI IO UUC II I 1 111 JC, I [ |UJ ill Hy pciyl l.ltxF lib u| ip I.UU 1 rli m l A / i f H r .
no interest is charged. The entry for the sale is
i. DcLdlloc dll pdyilltil lLo dlu UUU'Wlll III 1 [ ycdFj
■ ' Cash ! "'' . $ioo ; v - ■;
: : Installment receivable, Year 1 900 . ..
Cost of installment sales ;; :\V- y 600:-,f> :■ • • :.■ ■ -
• •. / ;■ Inventory : •; /■:- ' - $ 600
• Installment sales V : ' . 1,000
SU 1: External Financial Statements and Revenue Recognition 31
d. At the end of the period, a portion of deferred gross profit is realized.
■■■■
In December, when the first installment is received, the entry is '
. Cash : V $100 ' '
. Installment receivable, Year 1 $100 .. .
At December 31, installment sales and cost of installment sales are closed, and deferred gross profit is recognized.
Moreover; deferred gross profit must be adjusted to report the portion that has been earned. Given that $200 of the total
price has been received, $80 of the gross profit ($200 x 40%) has been earned. The entry is
Installment sales $1,000 ..
Cost of installment sales $600 . ■
Deferred gross profit -400'.
Deferred gross profit (Year 1) $80
. Realized gross profit $80
Net income should include only the $80 realized gross profit for the period. The balance sheet should report a receivable of
$800 minus the deferred gross profit of $320. Thus, the net receivable is $480.
Balance sheet:
.. . Installment receivable
. . (net of deferred gross profit of $320) $480
e. The gross profit percentage for the period of the sale continues to be applied to the
realization of deferred gross profit from sales of that period.
EXAMPLE
In Year 2, the remaining $800 is received, and the $320 balance of deferred gross profit is recognized. If only $400 were
received in Year 2 (if payments were extended), the December, Year 2, statements would report a $400 installment
receivable and $160 of deferred gross profit.
3. Cost-Recovery Method
a. The cost-recovery method may be used only in the same circumstances as the
installment method. However, no profit is recognized until collections exceed the cost
of the item sold. Subsequent receipts are treated entirely as revenues.
EXAMPLE
In Year 1, Creditor made a $100,000 sale. The cost of the item sold was $70,000, and Year 1 collections equaled $50,000.
In Year 2, collections equaled $25,000, and $10,000 of the receivable was determined to be uncollectible. The net
receivable (receivable ~ deferred profit) was $0 at the end of Year 2. The following entries are based on the cost-recovery
method: v . . . •
Yearl: : Receivable .
Inventory
Deferred gross profit
. $100,000
$70,000
30,000
Cash V; ^
Receivable
$50,000
$50,000
■ Year 2: - ■ Cash
Deferred gross profit. . .
Receivable
Realized gross profit
$25,000 .
5,ooo
$25,000
5,000
' Deferred gross profit
Receivable
: $10,000 .
$10,000 ..
32 SU 1: External Financial Statements and Revenue Recognition
4. Deposit Method
a. This method is used when cash is received, but the criteria for a sale have not been
met. Thus, the seller continues to account for the property in the same way as an
owner. No revenue or profit is recognized because it has not been earned, e.g., by
transferring the property. The entry is
Cash $XXX
Deposit liability $XXX
Stop and review! You have completed the outline for this subunit. Study multiple-choice
questions 27 and 28 on page 42.
1.7 REVENUE RECOGNITION - LONG-TERM CONSTRUCTION CONTRACTS
1. The Compieted-Contract Method
a. The completed-contract method is used to account for a long-term project when the
percentage-of-completion method is inappropriate.
1) It defers ail contract costs in the inventory account construction in progress
until the project is completed. It records progress billings in the contra-inventory
account progress billings.
2) Revenue and gross profit are recognized only upon completion.
2. The Percentage-of-Completion Method
a. Percentage-of-completion is the preferable method. It records (1) all contract costs in
construction in progress and (2) ail amounts billed in progress billings. However, the
percentage-of-completion method differs from the completed-contract method
because it recognizes revenue on long-term contracts when the
1) Extent of progress toward completion, contract revenue, and contract costs are
reasonably estimable;
2) Enforceable rights regarding goods or services to be provided, the consideration
to be exchanged, and the terms of settlement are clearly specified; and
3) Obligations of the parties are expected to be fulfilled.
b. The amount of gross profit recognized in a period is calculated as follows:
1) Calculate the estimated total gross profit on the project.
EXAMPLE
A contractor is constructing an office complex for a real estate developer. The agreed-upon contract price was $75 million.
As of the close of Year 4 of the project, the contractor had incurred $44 million of costs. By its best estimates as of that
aaie, costs remaining xo Tinisn tne project were i y million.
'■ • ■ Contract price .. ■' $75,000,000. ‘V ' .
Minus: costs incurred to date •• (44,000,000) . . ■ '
. Minus: estimated costs to complete ( i g . o o o . o o o ) : ^
Estimated total gross profit $12,000,000 .
2) Calculate the percentage of the project completed as of the reporting date,
determined by the ratio of costs incurred thus far to estimated total costs.
Total estimated costs for the project as of the end of Year 4 are calculated as follows:
Costs incurred to date $44,000,000
Estimated costs to complete ■ 19,000,000
Totai estimated costs .. $63,000,000 ..
The project is therefore 69.8% complete ($44,000,000 * $63,000,000).
SU 1: External Financial Statements and Revenue Recognition 33
3) Subtract the gross profit recognized so far.
EXAMPLE ; X
The contractor will recognize $2,151,000 in gross profit for Year 4, calculated as follows:
Estimated total gross profit $12,000,000
. . Times: percentage, complete , X 69.8%
Gross profit earned to date $ 8,376,000
Minus: gross profit recognized in prior periods (given) (6,225,000)
X ■ Gross profit for current period . $2,151,000
c. As soon as an estimated loss on any project becomes apparent, it is recognized in
full, under both the completed-contract and percentage-of-completion methods.
3. Comparative Journal Entries
V . : X ' EXAMPLE p..
A contractor agrees to build a bridge that will take 3 years to complete. The contract price is $2 million and expected total
costs arie $1.2 million. '
Year 1 . Year2 ; Year3
: Costs incurred during each year . . . $300,000. $600,000 $550,000
Costs expected in future " : 900,000 600,000 :. 0
By the end of Year 1, 25% ($300,000$1,200,000) of expected-costs has been incurred.. Using percentage-of-completion,.
the contractor, wi.li recognize 25% of the revenue or gross profit that will be earned on the project. The totai gross profit is
expected to be $800,000 ($2,000*000 - $1,200,000), so $200,000.($800,000-* 25%); of gross profit should be recognized in
Year 1. , ■■ :
Percentage-of-Complotion Completed-Contract
Year 1: Construction in progress $300,000 $300,000
Cash or accounts payable $300,000 $300,000
Construction in progress $200,000
Construction gross profit . . . $200,000 -
At the end of Year 2, total costs incurred, are $900,000 ($300,000 + $600,000). Given that $600,000. is expected to bo
incurred in. the future, the total expected cost is $1,500,000 ($900,000 + $600,000), and the estimate of gross profit is
$500,000 ($2,000,000 contract price - $1,500,000 costs). If the project is 60% complete ($900,000 * $1,500,000),
$300,000 of cumulative gross profit should be recognized for Years 1 and 2 ($500,000 * 60%) under percentage-ofcompletion.
Because $200,000 was recognized in Year 1, $100,000 should be.recognized in Year 2. y
" Percentage-of-Completion Completed-Contract .
Year 2: Construction in progress $600,000 $600,000 '
-/X: Cash or accounts payable ;V: . .. .. $600,000 $600,000 -Y.:
Construction in progress $100,000 : / ■ V; ■ • • ’ X
Construction gross profit $100,000 . V'- - .
At the end of the third year, total Costs are $1,450,000. Thus, the total gross profit is known to be $550,000, Because a total
of $300,000 was recognized in Years 1 and 2, $250,000 should be recognized in Year 3, using percentage-of-completion. *
X-c-X- ' Percentage-of-Completion Completed-Contract
Year 3: Construction in progress $550,000 $550,000
Cash or accounts payable . $550,000 $550,000
X X- 8 Cash ' . . ' $2,000,000 $2,000,000
Construction in progress $1,750,000 $1,450,000
Construction gross profit 250,000 550,000
* This is the first recognition of gross profit under the completed-contract method.
34 SU 1: External Financial Statements and Revenue Recognition
4. Progress Billings
a. Ordinarily, progress billings are made and payments are received during the term of
the contract. The entries are
The customer is billed:
Accounts receivable $XXX
Progress billings $XXX
The customer pays:
Cash $XXX
Accounts receivable $XXX
1) Neither billing nor the receipt of cash affects gross profit. Moreover, billing,
receipt of payment, and incurrence of cost have the same effects under both
accounting methods.
b. The difference between construction in progress (costs and recognized gross profit)
and progress billings to date is reported as a current asset if construction in
progress exceeds total billings and as a current liability if billings exceed
construction in progress.
Closing entry:
Progress billings $XXX
Construction in progress $XXX
IFRS Difference
The completed-contract method is not used. When the outcome of a
long-term construction contract cannot be reliably estimated, revenue
recognition is limited to recoverable costs incurred. Contract costs must be
recognized as an expense in the period in which they are incurred.
Stop and review! You have completed the outline for this subunit. Study multiple-choice
questions 29 and 30 on page 43.
QUESTIONS
1.1 Concepts of Financial Accounting
1. A primary objective of external financial reporting
is
A. Direct measurement of the value of a business
enterprise.
B. Provision of information that is useful to
present and potential investors, creditors, and
others in making rational financial decisions
regarding the enterprise.
C. Establishment of rules for accruing liabilities.
D. Direct measurement of the enterprise’s stock
price.
Answer (B) is correct.
REQUIRED: The primary objective of external financial
reporting.
DISCUSSION: According to the FASB’s Conceptual
Framework, the objectives of external financial reporting are to
provide information that (1) is useful to present and potential
investors, creditors, and others in making rational financial
decisions regarding the enterprise; (2) helps those parties in
assessing the amounts, timing, and uncertainty of prospective
cash receipts from dividends or interest and the proceeds from
sale, redemption, or maturity of securities or loans; and
(3) concerns the economic resources of an enterprise, the claims
thereto, and the effects of transactions, events, and
circumstances that change its resources and claims thereto.
Answer (A) is incorrect. Financial reporting is not designed to
measure directly the value of a business. Answer (C) is incorrect.
While rules for accruing liabilities are a practical concern, the
establishment of such rules is not a primary objective of external
reporting. Answer (D) is incorrect. The objectives of financial
accounting are unrelated to the measurement of stock prices;
stock prices are a product of stock market forces.
SU 1: External Financial Statements and Revenue Recognition 35
2. Notes to financiai statements are beneficial in
meeting the disclosure requirements of financial
reporting. The notes should not be used to
A. Describe significant accounting policies.
B. Describe depreciation methods employed by
the company.
C. Describe principles and methods peculiar to
the industry in which the company operates,
when these principles and methods are'
predominantly followed in that industry.
D. Correct an improper presentation in the
financial statements.
Answer (D) is correct.
REQUIRED: The improper use of notes in financial
statements.
DISCUSSION: Financial statement notes should not be used
to correct improper presentations. The financial statements
should be presented correctly on their own. Notes should be
used to explain the methods used to prepare the financial
statements and the amounts shown. The first footnote typically
describes significant accounting policies.
3. Which of the following is true regarding the
comparison of managerial and financial accounting?
A. Managerial accounting is generally more
precise.
B. Managerial accounting has a past focus, and
financial accounting has a future focus.
C. The emphasis on managerial accounting is
relevance, and the emphasis on financial
accounting is timeliness.
D. Managerial accounting need not follow
generally accepted accounting principles
(GAAP), while financial accounting must follow
them.
Answer (D) is correct.
REQUIRED: The true comparison of managerial and
financial accounting.
DISCUSSION: Managerial accounting assists management
decision making, planning, and control. Financial accounting
addresses accounting for an entity’s assets, liabilities, revenues,
expenses, and other elements of financial statements. Financial
statements are the primary method of communicating to external
parties information about the entity’s results of operations,
financial position, and cash flows. For general-purpose financial
statements to be useful to external parties, they must be
prepared in conformity with accounting principles that are
generally accepted in the United States. However, managerial
accounting information is primarily directed to specific internal
users. Hence, it ordinarily need not follow such guidance.
Answer (A) is incorrect. Managerial accounting may be no
more precise than financial accounting. For example, it relies on
allocations of certain product'costs (e.g., overhead) that are
based on assumptions rather than on cause-and-effect
relationships. Answer (B) is incorrect. Only managerial
accounting is forward-looking. Financial accounting essentially
records what has already occurred. Answer (C) is incorrect.
Choices of financial accounting information to be reported must
balance relevance (including timeliness) and reliability. External
parties will not find financial accounting information useful if it is
timely but unreliable.
1.2 Statement of Financial Position (Balance Sheet)
4. When classifying assets as current and noncurrent
for reporting purposes,
A. The amounts at which current assets are
carried and reported must reflect realizable
cash values.
B. Prepayments for items such as insurance or
rent are included in an “other assets” group
rather than as current assets as they will
ultimately be expensed.
C. The time period by which current assets are
distinguished from noncurrent assets is
determined by the seasonal nature of the
business.
Answer (D) is correct.
REQUIRED: The true statement about the classification of
current assets.
DISCUSSION: For financial reporting purposes, current
assets consist of cash and other assets or resources expected to
be realized in cash, sold, or consumed during the longer of
1 year or the normal operating cycle of the business.
Answer (A) is incorrect. Current assets are measured using
different attributes, for example, lower of cost or market for
inventory and net realizable value for accounts receivable.
Answer (B) is incorrect. Prepayments may qualify as current
assets. They often will be consumed during the operating cycle.
Answer (C) is incorrect. The classification criterion is based on
the normal operating cycle regardless of the seasonality of the
business.
D. Assets are classified as current if they are
reasonably expected to be realized in cash or
consumed during the normal operating cycle.
36 SU 1: External Financial Statements and Revenue Recognition
5. Abernathy Corporation uses a calendar year for
financial and tax reporting purposes and has
$100 million of mortgage bonds due on January 15,
Year 2. By January 10, Year 2, Abernathy intends to
refinance this debt with new long-term mortgage
bonds and has entered into a financing agreement
that clearly demonstrates its ability to consummate
the refinancing. This debt is to be
A. Classified as a current liability on the statement
of financial position at December 31, Year 1.
B. Classified as a long-term liability on the
statement of financial position at December 31,
Year 1.
C. Retired as of December 31, Year 1.
D. Considered off-balance-sheet debt.
Answer (B) is correct.
REQUIRED: The balance sheet treatment of maturing
long-term debt that is to be refinanced on a long-term basis.
DISCUSSION: Short-term obligations expected to be
refinanced should be reported as current liabilities unless the firm
both plans to refinance and has the ability to refinance the debt
on a long-term basis. The ability to refinance on a long-term
basis is evidenced by a post-balance-sheet date issuance of
long-term debt or a financing arrangement that will clearly permit
long-term refinancing.
Answer (A) is incorrect. The company intends to refinance
the debt on a long-term basis. Answer (C) is incorrect. The debt
has not been retired. Answer (D) is incorrect. The debt is on the
balance sheet.
6. A statement of financial position is intended to help
investors and creditors
A. Assess the amount, timing, and uncertainty of
prospective net cash inflows of a firm.
B. Evaluate economic resources and obligations
of a firm.
C. Evaluate economic performance of a firm.
D. Evaluate changes in the ownership equity of a
firm.
Answer (B) is correct.
REQUIRED; The purpose of a statement of financial
position.
DISCUSSION: The statement of financial position, or
balance sheet, provides information about an entity's resource
structure (assets) and financing structure (liabilities and equity) at
a moment in time. According to the FASB’s Conceptual
Framework, the statement of financial position does not purport
to show the value of a business, but it enables investors,
creditors, and other users to make their own estimates of value. It
helps users to assess liquidity, financial flexibility, profitability,
and risk.
Answer (A) is incorrect. Providing information to help assess
the amount, timing, and uncertainty of cash flows is an objective
of the statement of cash flows. Answer (C) is incorrect. The
primary focus of financial reporting is information about an
enterprise’s performance provided by measures of earnings and
its components. Hence, an income statement is more directly
useful to investors and creditors for evaluating economic
performance. Answer (D) is incorrect. Disclosures of changes in
shareholders’ equity, in either the basic statements, the notes
thereto, or a separate statement, help users to evaluate changes
in the ownership equity of a firm.
7. A statement of financial position provides a basis
for all of the following except
A. Computing rates of return.
B. Evaluating capital structure.
C. Assessing liquidity and financial flexibility.
D. Determining profitability and assessing past
performance.
Answer (D) is correct.
REQUIRED: The item for which a statement of financial
position is not useful.
DISCUSSION: The statement of financial position, also
known as the balance sheet, reports an entity’s financial position
at a moment in time. It is therefore not useful for assessing past
performance for a period of time. A balance sheet can be used to
help users assess liquidity, financial flexibility, and risk.
Answer (A) is incorrect. The statement of financial position
reports the amount of invested capital necessary for computing
rates of return. Answer (B) is incorrect. The statement of financial
position reports the details of an entity’s capital structure, i.e., the
levels of debt and equity. Answer (C) is incorrect. The statement
of financial position reports current and noncurrent assets and
liabilities. Thus, it is useful for assessing liquidity and financial
flexibility.
SU 1: External Financial Statements and Revenue Recognition 37
8. The purchase of treasury stock is recorded on the
statement of financial position as a(n)
A. Increase in assets.
B. Decrease in liabilities.
C. Increase in shareholders’ equity.
D. Decrease in shareholders’ equity.
Answer (D) is correct.
REQUIRED: The accounting for the purchase of treasury
stock.
DISCUSSION: The purchase of treasury stock is recorded
on the statement of financial position as a decrease in
shareholders’ equity.
Answer (A) is incorrect. Treasury stock cannot be reported
as an asset. Answer (B) is incorrect. Treasury stock cannot be
reported as an asset nor is it associated with liabilities.
Answer (C) is incorrect. The purchase of treasury stock is
recorded on the statement of financial position as a decrease in
shareholders’ equity.
1.3 Income Statement and Statement of Comprehensive Income
9. In a multiple-step income statement for a retail
company, all of the following are included in the
operating section except
A. Sales.
B. Cost of goods sold.
C. Dividend revenue.
D. Administrative and selling expenses.
Answer (C) is correct.
REQUIRED: The item excluded from the operating section of
a multiple-step income statement of a retailer.
DISCUSSION: The operating section of a retailer’s income
statement includes all revenues and costs necessary for the
operation of the retail establishment, e.g., sales, cost of goods
sold, administrative expenses, and selling expenses. Dividend
revenue, however, is classified under other revenues. In a
statement of cash flows, cash dividends received are considered
an operating cash flow.
Answer (A) is incorrect. Sales is part of the normal
operations of a retailer. Answer (B) is incorrect. Cost of goods
sold is part of the normal operations of a retailer. Answer (D) is
incorrect. Administrative and selling expenses are part of the
normal operations of a retailer.
10. When reporting extraordinary items,
A. Each item (net of tax) is presented on the face
of the income statement separately as a
component of net income for the period.
B. Each item is presented exclusive of any related
income tax.
C. Each item is presented as an unusual item
within income from continuing operations.
D. All extraordinary gains or losses that occur in a
period are summarized as total gains and total
losses, then offset to present the net
extraordinary gain or loss.
Answer (A) is correct.
REQUIRED: The true statement about the reporting of
extraordinary items.
DISCUSSION: Extraordinary items are reported net of tax
after discontinued operations.
Answer (B) is incorrect. Extraordinary items are to be
reported net of the related tax effect. Answer (C) is incorrect.
Extraordinary items are not reported in the continuing operations
section of the income statement. Answer (D) is incorrect. Each
extraordinary item is to be reported separately.
11. Which one of the following would be shown on a
multiple-step income statement but not on a singlestep
income statement?
A. Loss from discontinued operations.
B. Gross profit.
C. Extraordinary gain.
D. Net income from continuing operations.
Answer (B) is correct.
REQUIRED: The item on a multiple-step income statement
not shown on a single-step income statement.
DISCUSSION: A single-step income statement combines all
revenues and gains, combines all expenses and losses, and
subtracts the latter from the former in a “single step” to arrive at
net income. Gross profit, being the difference between sales
revenue and cost of goods sold, does not appear on a singlestep
income statement.
Answer (A) is incorrect. Loss from discontinued operations is
shown on both a muitiple-step and a single-step income
statement. Answer (C) is incorrect. Extraordinary gain is shown
on both a multiple-step and a single-step income statement.
Answer (D) is incorrect. Net income from continuing operations is
shown on both a multiple-step and a single-step income
statement.
38 SU 1: External Financial Statements and Revenue Recognition
12. The profit and loss statement of Madengrad
Mining includes the following information for the
current fiscal year:
Sales $160,000
Gross profit 48,000
Year-end finished goods inventory 58,300
Opening finished goods inventory 60,190
The cost of goods manufactured by Madengrad for
the current fiscal year is
A. $46,110
B. $49,890
C. $110,110
D. $113,890
Answer (C) is correct.
REQUIRED: The cost of goods manufactured.
DISCUSSION: Madengrad’s cost of goods manufactured
can be calculated as follows:
Sales $160,000
Less: gross profit (48,000)
Cost of goods sold $112,000
Add: ending finished goods 58,300
Goods available for sale $170,300
Less: beginning finished goods (60,190)
Cost of goods manufactured $110,110
Answer (A) is incorrect, improperly beginning with gross
profit instead of sales results in $46,110. Answer (B) is incorrect.
Improperly beginning with gross profit instead of sales, then
improperly subtracting ending finished goods and adding
beginning finished goods results in $49,890. Answer (D) is
incorrect. Improperly subtracting ending finished goods and
adding beginning finished goods results in $113,890.
13. Given the following data for Scurry Company,
what is the cost of goods sold?
Beginning inventory of finished goods
Cost of goods manufactured
Ending inventory of finished goods
Beginning work-in-process inventory
Ending work-in-process inventory
A. $500,000
B. $600,000
C. $800,000
D. $950,000
$100,000
700.000
200.000
300,000
50,000
Answer (B) is correct.
REQUIRED: The cost of goods sold.
DISCUSSION: Scurry's cost of goods sold can be calculated
as follows:
Beginning inventory of finished goods $ 100,000
Add: cost of goods manufactured 700,000
Less: ending inventory of finished goods (200,000)
Cost of goods sold $ 600.000
Answer (A) is incorrect. The amount of $500,000 results from
failing to include beginning finished goods inventory. Answer (C)
is incorrect. The amount of $800,000 results from failing to
subtract ending finished goods inventory. Answer (D) is incorrect.
The amount of $950,000 results from improperly including
work-in-process inventories.
14. Comprehensive income is best defined as
A. Net income excluding extraordinary gains and
losses.
B. The change in net assets for the period
including contributions by owners and
distributions to owners.
C. Total revenues minus total expenses.
D. The change in net assets for the period
excluding owner transactions.
Answer (D) is correct.
REQUIRED: The definition of comprehensive income.
DISCUSSION: Comprehensive income includes all changes
in equity of a business entity except those changes resulting from
investments by owners and distributions to owners.
Comprehensive income includes two major categories: net
income and other comprehensive income (OCI). Net income
includes the results of operations classified as income from
continuing operations, discontinued operations, and extraordinary
items. Components of comprehensive income not included in the
determination of net income are included in OCI, for example,
unrealized gains and losses on available-for-sale securities
(except those that are hedged items in a fair value hedge).
Answer (A) is incorrect. Net income excluding extraordinary
gains and losses is income before extraordinary items.
Answer (B) is incorrect. The change in net assets for the period
including contributions by owners and distributions to owners is
the change in equity. Answer (C) is incorrect. Total revenues
minus total expenses is operating income.
SU 1: External Financial Statements and Revenue Recognition 39
15. The financial statement that provides a summary
of the firm’s operations for a period of time is the
A. Income statement.
B. Statement of financial position.
C. Statement of shareholders’ equity.
D. Statement of retained earnings.
Answer (A) is correct.
REQUIRED: The financial statement that provides a
summary of the firm’s operations for a period of time.
DISCUSSION: The results of operations for a period of time
are reported in the income statement (statement of earnings) on
the accrual basis using an approach oriented to historical
transactions.
Answer (B) is incorrect. The statement of financial position
reports a firm’s resources and claims to those resources at a
moment in time. Answer (C) is incorrect. The statement of
shareholders' equity presents a reconciliation in columnar format
of the beginning and ending balances in the various equity
accounts. Answer (D) is incorrect. The statement of retained
earnings presents the changes in an entity’s retained earnings
during a period of time.
1.4 Statement of Changes in Equity and Equity Transactions
16. An appropriation of retained earnings by the
board of directors of a corporation for bonded
indebtedness will result in
A. The establishment of a sinking fund to retire
bonds when they mature.
B. A decrease in cash on the balance sheet with
an equal increase in the investment and funds
section of the balance sheet.
C. A decrease in the total amount of retained
earnings presented on the balance sheet.
D. The disclosure that management does not
intend to distribute assets, in the form of
dividends, equal to the amount of the
appropriation.
Answer (D) is correct.
REQUIRED: The effect of an appropriation of retained
earnings.
DISCUSSION: The appropriation of retained earnings is a
transfer from one retained earnings account to another. The only
practical effect is to decrease the amount of retained earnings
available for dividends. An appropriation of retained earnings is
purely for disclosure purposes.
Answer (A) is incorrect. The establishment of a sinking fund
is entirely independent of appropriating retained earnings.
Answer (B) is incorrect. Cash is unaffected. Answer (C) is
incorrect. The totai retained earnings will not change; however,
the total will appear as the sum of two retained earnings
accounts instead of one.
17. The statement of shareholders’ equity shows a
A. Reconciliation of the beginning and ending
balances in shareholders’ equity accounts.
B. Listing of all shareholders’ equity accounts and
their corresponding dollar amounts.
C. Computation of the number of shares
outstanding used for earnings per share
calculations.
D. Reconciliation of net income to net operating
cash flow.
Answer (A) is correct.
REQUIRED: The content of the statement of shareholders’
equity.
DISCUSSION: The statement of shareholders’ equity
(changes in equity) presents a reconciliation in columnar format
of the beginning and ending balances in the various
shareholders’ equity accounts. A statement of changes in equity
may include, for example, columns for (1) totals,
(2) comprehensive income, (3) retained earnings,
(4) accumulated OCI (but the components of OCI are presented
in another statement), (5) common stock, and (6) additional
paid-in capital.
Answer (B) is incorrect. A listing of all shareholders’ equity
accounts would provide little useful information. Answer (C) is
incorrect. The computation of the number of shares outstanding
used for EPS calculations is disclosed in the income statement.
Answer (D) is incorrect. The reconciliation of net income to net
operating cash flow is presented in the statement of cash flows.
40 SU 1: External Financial Statements and Revenue Recognition
18. Unless the shares are specifically restricted, a
holder of common stock with a preemptive right may
share proportionately in all of the following except
A. The vote for directors.
B. Corporate assets upon liquidation.
C. Cumulative dividends.
D. New issues of stock of the same class.
Answer (C) is correct.
REQUIRED: The item that is not a right of common
shareholders.
DISCUSSION: Common stock does not have the right to
accumulate unpaid dividends. This right is often attached to
preferred stock.
Answer (A) is incorrect. Common shareholders have the right
to vote (although different classes of shares may have different
privileges). Answer (B) is incorrect. Common shareholders have
the right to share proportionately in corporate assets upon
liquidation (but only after other claims have been satisfied).
Answer (D) is incorrect. Common shareholders have the right to
share proportionately in any new issues of stock of the same
class (the preemptive right).
19. On December 1, Noble Inc.'s Board of Directors
declared a property dividend, payable in stock held in
the Multon Company. The dividend was payable on
January 5. The investment in Multon stock had an
original cost of $100,000 when acquired 2 years ago.
The market value of this investment was $150,000 on
December 1, $175,000 on December 31, and
$160,000 on January 5. The amount to be shown on
Noble’s statement of financial position at
December 31 as property dividends payable would be
Answer (B) is correct.
REQUIRED: The amount of property dividends payable on
Noble's statement of financial position.
DISCUSSION: When a property dividend is declared, the
property is remeasured at its fair value as of the declaration date.
This amount is then reclassified from retained earnings to
property dividends payable.
A. $100,000
B. $150,000
C. $160,000
D. $175,000
20. Which one of the following statements regarding
dividends is correct?
A. A stock dividend of 15% of the outstanding
common shares results in a debit to retained
earnings at the par value of the stock
distributed.
B. At the declaration date of a 30% stock
dividend, the carrying value of retained
earnings will be reduced by the fair market
value of the stock distributed.
C. The declaration of a cash dividend will have no
effect on book value per share.
D. The declaration and payment of a 10% stock
dividend will result in a reduction of retained
earnings at the fair market value of the stock.
Answer (D) is correct.
REQUIRED: The correct statement regarding dividends.
DISCUSSION: When a small stock dividend is declared (less
than 20% to 25% of the previously outstanding common shares),
retained earnings is debited for the fair value of the stock.
Answer (A) is incorrect. When a small stock dividend is
declared (less than 20% to 25% of the previously outstanding
common shares), retained earnings is debited for the fair value of
the stock. Answer (B) is incorrect. When a large stock dividend is
declared (more than 20% to 25% of the previously outstanding
common shares), retained earnings is debited for the par value of
the stock. Answer (C) is incorrect. When a cash dividend is
declared, a portion of retained earnings is reclassified as a
current liability.
1.5 Statement of Cash Flows
21. When preparing the statement of cash flows,
companies are required to report separately as
operating cash flows all of the following except
A. Interest received on investments in bonds.
B. Interest paid on the company’s bonds.
C. Cash collected from customers.
D. Cash dividends paid on the company’s stock.
Answer (D) is correct.
REQUIRED: The item not reported separately as an
operating cash flow on a statement of cash flows.
DISCUSSION: In general, the cash flows from transactions
and other events that enter into the determination of income are
to be classified as operating. Cash receipts from sales of goods
and services, from interest on loans, and from dividends on
equity securities are from operating activities. Cash payments to
suppliers for inventory; to employees for wages; to other
suppliers and employees for other goods and services; to
governments for taxes, duties, fines, and fees; and to lenders for
interest are also from operating activities. However, distributions
to owners (cash dividends on a company’s own stock) are cash
flows from financing, not operating, activities.
SU 1: External Financial Statements and Revenue Recognition 41
22. All of the following should be classified under
the operating section in a statement of cash flows
except a
A. Decrease in inventory.
B. Depreciation expense.
C. Decrease in prepaid insurance.
D. Purchase of land and building in exchange for
a long-term note.
Answer (D) is correct.
REQUIRED: The item not classified as an operating item in a
statement of cash flows.
DISCUSSION: Operating activities include all transactions
and other events not classified as investing and financing
activities. Operating activities include producing and delivering
goods and providing services. Cash flows from such activities are
usually included in the determination of net income. However, the
purchase of land and a building in exchange for a long-term note
is an investing activity. Because this transaction does not affect
cash, it is reported in related disclosures of noncash investing
and financing activities.
23. Kelli Company acquired land by assuming a
mortgage for the full acquisition cost. This transaction
should be disclosed on Kelli's statement of cash flows
as a(n)
A. Financing activity.
B. Investing activity.
C. Operating activity.
D. Noncash financing and investing activity.
Answer (D) is correct.
REQUIRED: The classification of an acquisition of land by
assuming a mortgage.
DISCUSSION: The exchange of debt for a long-lived asset
does not involve a cash flow. It is therefore classified as a
noncash financing and investing activity.
Answer (A) is incorrect. To be classified as a financing
activity, the transaction must have involved a cash flow.
Answer (B) is incorrect. To be classified as an investing activity,
the transaction must have involved a cash flow. Answer (C) is
incorrect. To be classified as an operating activity, the transaction
must have involved a cash flow.
24. All of the following should be classified as
investing activities in the statement of cash flows
except
A. Cash outflows to purchase manufacturing
equipment.
B. Cash inflows from the sale of bonds of other
entities.
C. Cash outflows to lenders for interest.
D. Cash inflows from the sale of a manufacturing
plant.
Answer (C) is correct.
REQUIRED: The cash flow not from an investing activity.
DISCUSSION: Investing activities include the [ending of
money and the collecting of those loans; the acquisition, sale, or
other disposal of debt or equity instruments; and the acquisition,
sale, or other disposition of assets (excluding inventory) that are
held for or used in the production of goods or services. Investing
activities do not include acquiring and disposing of certain loans
or other debt or equity instruments that are acquired specifically
for resale. Cash outflows to lenders for interest are cash from an
operating, not an investing, activity.
Answer (A) is incorrect. The purchase of equipment is an
investing activity. Answer (B) is incorrect. The sale of bonds
issued by another entity is an investing activity. Answer (D) is
incorrect. The sale of a plant is an investing activity.
25. To calculate cash flows using the indirect
method, which one of the following items must be
added back to net income?
A. Revenue.
B. Marketing expense.
C. Depreciation expense.
D. Interest income.
Answer (C) is correct.
REQUIRED: The components of the indirect method of
stating operating cash flows.
DISCUSSION: The indirect method begins with
accrual-basis net income or the change in net assets and
removes items that did not affect operating cash flow.
Depreciation is a non-cash item and thus does not affect the
cash flows. This amount must be added back to net income
because it decreased net income even though it had no cash
effect.
Answer (A) is incorrect. Revenues are not added back to net
income when using the indirect method of cash flows. Revenues
affect cash flows. Answer (B) is incorrect. Marketing expenses
are not added back to net income when using the indirect method
of cash flows. Marketing expenses affect cash flows. Answer (D)
is incorrect. Interest income is not added back to net income
when using the indirect method of cash flows. Interest income
affects cash flows.
42 SU 1: External Financial Statements and Revenue Recognition
26. Zip Company entered into the following
transactions during the year:
• Purchased stock for $200,000
® Purchased electronic equipment for use on the
manufacturing floor for $300,000
& Paid dividends to shareholders of Zip Company in
the amount of $800,000
The amount to be reported in the investing activities
section of Zip’s statement of cash flows would be
A. $200,000
B. $500,000
C. $800,000
D. $1,300,000
Answer (B) is correct.
REQUIRED: The amount to be reported in the investing
activities section of the statement of cash flows.
DISCUSSION: The statement of cash flows classifies an
enterprise’s cash flows into three categories. Investing activities
typically include the purchase and sale of securities of other
entities and the purchase and sale of property, plant, and
equipment. Thus, the amount to be reported in the investing
activities section of Zip’s statement of cash flows is $500,000
($200,000 + $300,000).
Answer (A) is incorrect. The amount of $200,000 results from
failing to include the purchase of manufacturing equipment.
Answer (C) is incorrect. The amount of $800,000 is the cash
flows from financing activities. Answer (D) is incorrect. The
amount of $1,300,000 results from improperly including the
dividend payment.
1.6 Revenue Recognition - Revenue Recognition after Delivery
27. Revenues of an entity are usually measured by
the exchange values of the assets or liabilities
involved. Recognition of revenue does not occur until
A. The revenue is realizable.
B. The revenue is realized and earned.
C. Products or services are exchanged for cash
or claims to cash.
D. The entity has substantially accomplished what
it agreed to do.
Answer (B) is correct.
REQUIRED: The appropriate timing of the recognition of
revenue.
DISCUSSION: According to the FASB’s conceptual
framework, revenues should be recognized when they are
realized or realizable and earned. Revenues are realized when
products, merchandise, or other assets are exchanged for cash
or claims to cash. Revenues are realizable when related assets
received or held are readily convertible to known amounts of
cash or claims to cash. Revenues are earned when the entity has
substantially accomplished what it must do to be entitled to the
benefits represented by the revenues.
Answer (A) is incorrect. Revenue also must be earned.
Answer (C) is incorrect. Exchange for cash or claim to cash does
not suffice for revenue recognition. Answer (D) is incorrect.
Revenue also must be realizable.
28. Robin Gavaskar, who recently founded a
company that produces baseball bats and balls,
wants to determine her company’s policy for revenue
recognition. According to the revenue recognition
principle, the most appropriate time to recognize
revenue would be when
A. The sale occurs.
B. Cash is received.
C. Production is completed.
D. Quarterly financial statements are prepared.
Answer (A) is correct.
REQUIRED: The most appropriate time to recognize
revenue.
DISCUSSION: Revenues are normally recognized when
they are realized or realizable and earned. Revenues are
realized (or realizable) when goods or services have been
exchanged for cash or claims to cash (assets readily convertible
to cash). Revenues are earned when the earning process is
substantially complete, and the entity is entitled to the resulting
benefits or revenues. The revenue recognition criteria are
ordinarily met at the point of sale (time of delivery of goods or
services).
Answer (B) is incorrect. Under the accrual basis of
accounting, revenue is not necessarily recognized when cash is
received. Answer (C) is incorrect. The criteria for revenue
recognition ordinarily have not been met until the product is sold.
Answer (D) is incorrect. Under the accrual basis of accounting,
revenues are normally recognized when they are realized or
realizable and earned, regardless of when the financial
statements are prepared.
SU 1: External Financial Statements and Revenue Recognition 43
1.7 Revenue Recognition -- Long-Term Construction Contracts
29. Paulson Company uses the percentage-ofcompletion
method to account for long-term
construction contracts. The following information
relates to a contract that was awarded at a price of
$700,000. The estimated costs were $500,000, and
the contract duration was 3 years.
Year 1 Year 2 Year 3
Cumulative cost to date $300,000 $390,000 $530,000
Costs to complete at
year end 250,000 130,000 __
Progress billings 325,000 220,000 155,000
Collections on account 300,000 200,000 200,000
Assuming that $65,000 was recognized as gross
profit in Year 1, the amount of gross profit Paulson
recognized in Year 2 was
A. $35,000
B. $70,000
C. $135,000
D. $170,000
Answer (B) is correct.
REQUIRED: The amount of gross profit.
DISCUSSION: Determining the annual recognized gross
profit requires calculation of the estimated total gross profit.
Year 1 Year 2
Contract price $700,000 $700,000
Minus: estimated total costs
Costs to date $300,000 $390,000
Estimated costs to complete 250,000 130,000
Estimated total costs $550,000 $520,000
Estimated total gross profit $150,000 $180,000
The completion percentage for Year 2 is the ratio of costs
incurred to date to estimated total costs ($390,000 * $520,000 =
75%). The cumulative gross profit recognized at the end of
Year 2 is therefore $135,000 ($180,000 x 75%). Because
$65,000 was recognized in Year 1, the amount recognized in
Year 2 is $70,000 ($135,000 - $65,000).
Answer (A) is incorrect. The amount of $35,000 is the gross
profit recognized in Year 3. Answer (C) is incorrect. The amount
of $135,000 is the cumulative gross profit recognized at the end
of Year 2, Answer (D) is incorrect. The amount of $170,000 is the
total gross profit recognized over the term of the contract.
30. A company appropriately uses the completedcontract
method to account for a long-term
construction contract. Revenue is recognized when
progress billings are
Recorded Collected
A. No Yes
B. Yes Yes
C. Yes No
D. No No
Answer (D) is correct,
REQUIRED: The effect of progress billings on the
recognition of revenue.
DISCUSSION: GAAP require that revenue be recognized
when it is realized or realizable and earned. Linder the
completed-contract method, revenue recognition is appropriate
only at the completion of the-contract. Neither the recording nor
the collection of progress billings affects this recognition.
Use Gieim CMA Test Prep for interactive testing with additional multiple-choice questions!
44 SU 1: External Financial Statements and Revenue Recognition
1.8 ESSAY QUESTIONS
Scenario for Essay Questions 1. 2
Ambyt, Inc., a manufacturer of high-value integrated control devices, became a publicly owned
company through an initial public offering less than 2 years ago. The company had been a privately
held firm for over 15 years and has retained its senior management team. The CEO recommended to
the CFO that they hire an assistant to prepare the additional reports required of a public company
rather than continuing to rely on the outside accounting firm that has been preparing them since the
IPO. Wayne Grant, who has experience preparing SEC filings, was hired 6 months ago to fill this role
and reports to the CFO. On July 3, Grant prepared the quarterly reports for the period ending June 30,
with information from the Sales and Accounting Departments. Ambyt treats sales and administrative
expenses as period expenses; these expenses average about 14% of sales. Parts of the statements
are shown below.
Income Statement for Period Ending June 30
Sales $14,321,000
Less returns and allowances 128,000
Net sales $14,193,000
Cost of goods sold 9,651,000
Gross profit $ 4,542,000
Selling & administrative expenses 2,024,000
Income from operations $ 2,518,000
Partial Balance Sheet as of June 30
Current assets
Cash $ 269,419
Accounts receivable 2,278,444
Notes receivable 558,000
Inventories 896,000
Short-term investments 532,000
Prepaid expenses 24,222
Supplies 58,798
Total current assets $4,616,883
Current liabilities
Accounts payable $1,639,000
Notes payable 580,000
Accrued wages 421,000
Taxes payable 187,000
Other liabilities 66,000
Total current liabilities $2,893,000
On July 4, Grant learned from the shipping supervisor that a large order of control devices
scheduled to be shipped on June 28 would not be ready until July 6 due to an unauthorized work
stoppage by the production machinists. Later in the same day, Grant learned that the manager for the
Sales Department had included the sale in the June 30 report because the work stoppage was not
authorized by the machinist union and therefore was beyond their control. The revenue reported for this
sale was $1,250,000, with an associated cost of goods of $715,000.
SU 1: External Financial Statements and Revenue Recognition 45
With this information, Grant determined that he should change the reports he prepared for the
period. He discussed this situation with the CFO. The CFO refused to consider the change, explaining
that consistent earnings growth is a primary driver of share price, and with Ambyt shares trading at a
P/E of 22, the share price would likely fall even though there was no real problem with production. The
CFO stated that although production delays were not common, they had occasionally occurred
throughout his years with Ambyt and that it was not a “big deal.”
Questions
1. Should the large shipment originally scheduled for June 28 be included in Ambyt’s June 30
Income Statement? Explain your answer.
2. Assuming Grant revises the reports to exclude the sale from the period,
a. Calculate the revised income from operations for the period ending June 30
b. Describe the changes that would be made to Ambyt’s June 30 balance sheet
c. Explain how the revisions will impact Ambyt’s cash flow statement
46 SU 1: External Financial Statements and Revenue Recognition
Essav Questions 1.2 — Unofficial Answers
1. Revenue for a manufacturing company is usually properly recognized when the product is
delivered. The product was not delivered during the period, and the revenue cannot be
recognized during the period. The reason for the delay in delivery has no bearing on the
timing of the revenue recognition.
2. a. The entry for sales will be decreased by the $1,250,000 sale amount. The entry for
cost of goods sold will decrease by $715,000, resulting in a net adjustment of
$(535,000) to the income from operations entry. The income from operations will be
$1,982,400. The selling expenses and administration expenses are period expenses
and are not affected by the timing of the sale.
b. Accounts receivable will decrease by the amount of the sale to $1,028,444. Inventories
will rise by only the cost of goods sold amount of $715,000, not the full sales amount.
c. The income statement change will be accompanied by changes to the inventory and
accounts receivable entries on the balance sheet. The cash flow statement entries for
changes in inventory and changes in accounts receivable will be altered (as will the
net income entry if using the indirect method to construct the statement). Entries to
income taxes payable will also be altered.
practice additional essa;
Need more?
G leim offers even more help to CMA candidates from-experienced professors
gf LIVE review and study sessions
0f Professor leadership
0 Interactive LIVE community
Visit gleim.com/MoreCMA or call 800.874.5346 to find locations near you.
STUDY U N I T TWO
MEASUREMENT, VALUATION, AND DISCLOSURE:
INVESTMENTS AND SHORT-TERM ITEMS
(23 pages of outline)
47
2.1 Accounts Receivable .................... .......................... ............................................................... 47
2 2 Inventory - Fundamentals . . . . . . > ............................. . . . . . . . . . . . . . . . . . . . 51
2.3 Inventory -- Cost Flow-Methods. . .:. ............................. . . . . . . ....................... . .... .... . . ... . . 55
. 2.4 . Inventory Measurement in the Financial Statements -- Lower of Cost or Market . . . . . . . . . 59
2.5 Classification of Investments................................... . . . .............. . . . . . . . . . . . . 60
2.6 Equity Method...................... . . . . . . . . . . . . . . . . . . ■ ... . .. .................. ................... .. . . . . . 64
2.7 Business Combinations and Consolidated Financial Statements ....................................... . 65
2.8 Different Types of Expenses and Liabilities............................................. .......................67
2.9 Essay Questions . . . ................ . . ............................................................................. . . . 81
This study unit is the second of three on external financial reporting decisions. The relative
weight assigned to this major topic in Part 1 of the exam is 15%. The three study units are
Study Unit 1: External Financial Statements and Revenue Recognition
Study Unit 2: Measurement, Valuation, and Disclosure: Investments and Short-Term Items
Study Unit 3: Measurement, Valuation, and Disclosure: Long-Term Items
2.1 ACCOUNTS RECEIVABLE
1. Overview
a. Accounts receivable, often called trade receivables, are the amounts owed to an entity
by its customers.
b. The recording of a receivable, which often coincides with revenue recognition, is
consistent with the accrual method of accounting.
c. Receivables should be separated into current and noncurrent portions. Most of the
entity’s accounts receivable are classified as current assets because they are
expected to be collected within 1 year or the entity’s normal operating cycle.
1) Current accounts receivable are reported in the balance sheet at net realizable
value (NRV), i.e., net of allowance for uncollectible accounts, allowance for
sales returns, and billing adjustments.
Gross accounts _ Allowance for _ NRV of
receivable uncollectible accounts accounts receivable
2) Noncurrent receivables are measured at net present value of future cash flows.
d. The direct write-off method expenses bad debts when they are determined to be
uncollectible. It is not acceptable under GAAP because it does not match revenue
and expense when the receivable and the write-off are recorded in different periods.
But this method is used for tax purposes.
2. Allowance for Customers’ Right of Sales Return
a. A provision must be made for the return of merchandise because of product defects,
customer dissatisfaction, etc.
b. To be consistent with the matching principle (recognition of revenue and related
expense in the same accounting period), the revenue from the sale of goods and the
expense for the estimated sales returns must be recognized on the same date.
1) Accordingly, an allowance for sales returns should be established.
48 SU 2: Measurement, Valuation, and Disclosure: Investments and Short-Term Items
EMMELB ;rv v
A company has $500,000 of sales in July, its first month of operations. Management estimates that total returns will be 1%
of sales. '
Recognition of revenue from sale Recognition of allowance for sales returns •
Cash/accounts receivable $500,000. . Sales returns (contra revenue) $5,000
■ Sales $500,000 Allowance for sales returns (contra asset) $5,000
3. Allowance for Uncollectible Accounts and Bad Debt Expense
a. Because collection in full of all accounts receivable is unlikely, the allowance for
uncollectible accounts must be recognized. This method attempts to match bad debt
expense with the related revenue.
b. The principal measurement issue for accounts receivable is the estimation of net
realizable value for balance sheet reporting and the related uncollectible accounts
expense (bad debt expense) for the income statement.
c. The bad debt expense recognized for the period increases the allowance for
uncollectible accounts. The allowance for uncollectible accounts is a contra account
to accounts receivable. Thus, the recognition of bad debt expense decreases the
balance of accounts receivable.
d. The two most common methods of measuring bad debt expense and the allowance for
uncollectible accounts are the percentage-of-sales method (an income statement
approach) and the percentage-of-receivables method (a balance sheet
approach). Both approaches have the goal of measuring accounts receivable at net
realizable value.
4. Income Statement Approach (Percentage of Sales)
a. The income statement approach calculates bad debt expense as a percentage of
credit sales reported on the income statement.
EXAMPLE.:;- : /M
A company’s year-end unadjusted trial balance reports the following amounts:
• Gross accounts receivable $100,000 Dr
Allowance for uncollectible accounts (year-beginning balance) - 1,000 Cr.
Sales on credit. . 250,000 Cr
According to past experience, 1% of the company's credit sales have been uncollectible. The company uses the income,
statement approach to calculate bad debt expense.
The bad debt expense recognized for the year is $2,500 ($250,000 * 1 %). The company records the following adjusting
journal entry: . . . . . . .
. ... S . Bad debt expense $2,500
Allowance.for uncollectible accounts $2,500
The total adjusted balances of allowance for uncollectible accounts and bad debt expense are $3,500 ($1,000 + $2,500)
and $2,500, respectively. The company reports net accounts receivable of $96,500 ($100,000 - $3,500) in its balance
sheet and bad debt expense of $2,500 in its statement of income. '
Balance sheet presentation: . • ' • . ' ' ;
Accounts receivable, net of allowance for uncollectible accounts of $3,500 $96,500.
5. Balance-Sheet Approach (Percentage of Receivables)
a. The balance sheet approach estimates the balance that should be recorded in the
allowance based on the collectibility of ending gross accounts receivable. Bad debt
expense is the amount necessary to adjust the allowance.
SU 2: Measurement, Valuation, and Disclosure: Investments and Short-Term items 49
EXAMPLE ■
■Using the data from the previous example, assume that the company uses the balance-sheet approach and that based on
the company’s experience, 6% of accounts receivable are determined to be uncollectible.
Thus, the ending balance of the allowance for uncollectible accounts is $6,000 ($100,000 * 6%). Because the allowance
currently has a balance of $1,000, the following journal entry is required:
Bad debt expense ($6,000 - $1,000) $5,000
Allowance for uncollectible accounts $5,000
Balance sheet presentation:
Accounts receivable, net of allowance for uncollectible accounts of $6,000 $94,000
b. An entity rarely has a single rate of uncollectibility for all accounts. Thus, an entity
using the balance sheet approach generally prepares an aging schedule for
accounts receivable.
■ EXAMPLE
Using the data from the previous example, assume that the company uses the following aging schedule to determine the
ending balance of tho allowance for uncollectible accounts:
Aging Interval Balance . : - Estimated Uncollectible . Ending Ailowance
Less than 30 days $70,000 2% $1,400
30-60 days 18,000 5% 900
61-90 days . 10,000 13% 1,300
Over 90 days. 2,000 20% 400
Tota! $100,000 $4,000
Thus, the ending balance of the allowance.for uncollectible accounts is $4,000. Because the allowance currently has a
balance of $1,000, the following journal ontry is required:
. . .. Bad debt expense ($4,000 - $1,000) $3,000
Allowance for uncollectibio accounts $3,000
Balance sheet presentation:
Accounts receivable, net of allowance for uncollectible accounts of $4,000 $96,000
6. Write-Off of Accounts Receivable
a. Some customers are unwilling or unable to satisfy their debts. A write-off of a specific
debt is recorded as follows:
Allowance for uncollectible accounts $XXX
Accounts receivable $XXX
1) Thus, the write-off of a particular bad debt has no effect on expenses.
2) Write-offs do not affect the carrying amount of net accounts receivable because
the reductions of gross accounts receivable and the allowance are the same.
Thus, they also have no effect on working capital.
b. Occasionally, a customer pays on an account previously written off.
Cash $xxx
Allowance for uncollectible accounts $XXX
1) Bad debt expense is not affected when an account receivable is written off or
when an account previously written off becomes collectible.
50 SU 2: Measurement, Valuation, and Disclosure: Investments and Short-Term Items
The following equation illustrates the reconciliation of the beginning and ending balances of the allowance for
uncollectible accounts:
Beginning allowance for uncollectible accounts
Plus: Bad debt expense recognized for the period
Less: Accounts receivable written off
Plus: Collection of accounts receivable previously written off
Ending allowance for uncollectible accounts____________
Under the income statement approach, bad debt expense is a percentage of sales on credit, and the ending
balance of the allowance is calculated using the equation above.
Under the balance sheet approach, the ending balance of the allowance is a percentage of the ending
balance of accounts receivable, and bad debt expense is calculated using the equation above.
7. Factoring of Accounts Receivable
a. Factoring is a transfer of receivables to a third party (a factor) who assumes the
responsibility of collection.
b. Factoring discounts receivables on a nonrecourse, notification basis. Thus, payments
by the debtors on the transferred assets are made to the factor. If the transferor
(seller) surrenders control, the transaction is a sale.
1) If a sale is without recourse, the transferee (credit agency) assumes the risks
and receives the rewards of collection. This sale is final, and the seller has no
further liabilities to the transferee. Accordingly, the receivables are no longer
reported on the seller’s books.
2) if a sale is with recourse, the transferor (seller) may be required to make
payments to the transferee or to buy back receivables in specified
circumstances. In this circumstance the transfer might not qualify as a sale. The
parties account for the transaction as a secured borrowing with a pledge of
noncash collateral. Accordingly, the receivables are still on the seller’s books
and it must recognize a liability for the amount of cash received.
EXAMPLE
A factor charges a 2% fee plus, an interest rate.of 18% on all cash advanced to a transferor of accounts receivable. Monthly.
sales are $100,000, and the factor advances 90% of the receivables submitted after deducting, the 2% fee and the interest.'/
Credit terms are net 60 days. What is the cost to the transferor of this arrangement?
:: . Amount of receivables submitted $100,000
Minus: 10% reserve • . (10,000)
Minus: 2% factor’s fee (2,000) :
Amount accruing to the transferor
. ' . .;.K Minus: 18% interest for 60 days
q> oo,UUU , - v - .
(2,640) [$88,000 * 18% * (60 - 360)]
Amount to be received immediately $ 85,360 •= • 7
The transferor also will receive the $10,000 reserve at the end of the 60-day period if it has not been absorbed by sales
returns and allowances. Thus, the total cost to the transferor to factor the receivables for the month is $4,640 ($2,000 factor
fee + interest of $2,640). Assuming that the factor has approved the customers’ credit in advance (the sale is without
recourse! the transferor will not absorb anv bad debtis.
The journal entry to record the preceding transaction is .
; . . Cash • . :: $85,360 .. '
Due from factor . :v ;x 10,000 V:v. : •..••• ■
. Loss on sale of receivables ■ 2,000 v 1' - 'v ■
. .. Prepaid interest • 2,640 v : . ..v
' Accounts receivable , .V - $100,000 - - V .; -;
SU 2: Measurement, Valuation, and Disclosure: Investments and Short-Term Items 51
c. The main reasons for factoring transactions are as follows:
1) A factor usually receives a high financing fee plus a fee for collection.
Furthermore, the factor often operates more efficiently than its clients because
of the specialized nature of its services.
2) An entity (seller) that uses a factor tries to speed up its collections. Also, it can
eliminate its credit department and accounts receivable staff. In addition, bad
debts are eliminated from the financial statements. These reductions in costs
can offset the fee charged by the factor.
d. Credit card sales are a common form of factoring. The retailer benefits by prompt
receipt of cash and avoidance of bad debts and other costs. In return, the credit card
company charges a fee.
Stop and review! You have compieted the outline for this subunit. Study multiple-choice
questions 1 through 5 beginning on page 70.
2.2 INVENTORY - FUNDAMENTALS
1. Overview
a. Inventory is the total of tangible personal property
1) Held for sale in the ordinary course of business,
2) In the form of work-in-process to be completed and sold in the ordinary course of
business, or
3) To be used up currently in producing goods or services for sale.
a) Inventory does not include long-term assets subject to depreciation.
b. Inventories are generally classified as current assets in the financial statements. They
are expected to be realized in cash or sold or consumed during the normal operating
cycle of the business.
1) Long-term assets either subject to depreciation or retired from regular use and
held for sale are not classified as inventories.
c. The inventories of a retailer (trading entity) consist of goods purchased to be resold
without substantial modification.
d. The inventories of a manufacturer consist of (1) goods to be consumed in production
(materials), (2) goods in the process of production (work-in-process), and (3) finished
goods.
e. The gross profit for the period is measured and presented in the income statement as
the difference between the amount of sales and the amount of cost of goods sold.
1) The calculation of cost of goods sold for a retailer and a manufacturer is
discussed in Study Unit 1, Subunit 3.
2. Cost Basis of Inventory - Initial Measurement
a. The cost of inventory includes all costs incurred in bringing the inventories to their
existing location and ready-to-use condition.
b. The cost of purchased inventories includes
1) The price paid or consideration given to acquire the inventory (net of trade
discounts, rebates, and other similar items);
2) import duties and other unrecoverable taxes; and
3) Handling, insurance, freight-in, and other costs directly attributable to
(a) acquiring finished goods and materials and (b) bringing them to their present
location and condition (salable or usable condition).
52 SU 2: Measurement, Valuation, and Disclosure: Investments and Short-Term Items
c. The cost of manufactured inventories (work-in-process and finished goods)
includes the costs of direct materials used and conversion costs. Conversion costs
consist of
1) Direct labor costs and
2) Manufacturing overhead costs.
3. Inventory Accounting Systems
a. A perpetual inventory system updates inventory accounts after each purchase or
sale. This system is generally more suitable for entities that sell relatively expensive
and heterogeneous items and requires continuous monitoring of inventory and cost of
goods sold accounts. Automobile dealers are an example. Under this system,
1) Purchases and other items related to inventory costing are charged directly to
inventory.
2) Inventory and cost of goods sold are adjusted as sales occur.
b. An advantage of the perpetual inventory system is that the amount of inventory on
hand and the cost of goods sold can be determined at any time. A disadvantage of
the perpetual inventory system is that the bookkeeping is more complex and
expensive.
c. In the periodic inventory system, inventory and cost of goods sold are updated at
specific intervals, such as quarterly or annually, based on the results of a physical
count. Bookkeeping is simpler under this system. Thus, entities with relatively
inexpensive and homogeneous items, such as grain dealers, that have no need to
continuously monitor their inventory and cost of goods sold generally use this system.
Under the periodic system,
1) Goods bought from suppliers and other items related to inventory costs are
tracked during the period in a separate temporary account (purchases).
2) The beginning inventory balance remains unchanged until the end of the period
when the purchases account is closed.
3) Changes in inventory and cost of goods sold are recorded only at the end of
the period, based on the physical count
^ e x a m p L e
Entity A’s January 1, Year 1, inventory consists of 1,000 units with a cost of $5 per unit/ The following are Entity A’s Year 2
transactions:.. • ■ .. .
April 1: sold 600 inventory units for $4,800 in cash. .
May 1: purchased 250 inventory: units: for $5 in cash per unit
The year-end result of the physical count Was 650 inventory units. The following are Entity A’s journal entries under the
perpetual and periodic systems: - : : ; ' • : • • .
Perpetual System Periodic System
Inventory sale April 1:
Cash .-v-."''.:
.. .. Sales ;■
$4,800 .
. $4,800
Cash V'v’ ' ;':
Sates
.. $4,800
$4,800 ;
: Cost of goods, sold (600 x $5)
Inventory
$3,000 . ,
$3,000
Inventory purchase May 1:
Inventory (250 x $5)
Cash \ .
$1,250
. , : $1,250 .
Purchases
Cash
$1,250
$1,250
- Continued on next page -
SU 2: Measurement, Valuation, and Disclosure: Investments and Short-Term items 53
EXAMPLE - continued
Perpetual System Periodic System
After the physical count on December 31:
No journal entry is needed because the physical count
equals the amount of inventory on the books
(1,000 - 600 + 250 - 650),
Inventory (year-end) (650 * $5)
Cost of goods sold (difference)
Inventory (beginning)
Purchases
$3,250
3,000
$5,000
1,250
Beginning inventory .
Purchases of inventory .during the period
. Ending inventory
Cost of goods sold
$5,000
1,250 ■
(3,250) , ■
$3,000
The perpetual and periodic systems have the same result. However, under the periodic system, the amounts of inventory
and cost of goods sold are updated only at the end of the period after the physical count.
4. Inventory Period-End Physical Count
a. An annual period-end inventory physical count is necessary under both the
perpetual and periodic inventory accounting systems. The amount of inventory
reported in the annua! financial statements should be based on a physical count.
1) Under the perpetual system, a physical count helps to detect (a) misstatements
in the records and (b) thefts of inventory. The differences between the physical
count and the inventory in the books (inventory shortages and overages) are
recognized as a separate line item in the current period income statement.
2) Under the periodic system, the amounts of inventory and cost of goods sold
can be determined based only on the results of a physical count. Thus, the
amount of cost of goods sold for the period includes both (a) inventory cost of
goods sold and (b) inventory shortages and overages.
b. For a physical count to be accurate, the entity must count all items considered to be
inventory and eliminate all items that are not. Items to be counted as inventory
include the following:
1) Goods in transit - Items in transit are inventories that on the physical count
date (a) are not on the entity’s premises and are on the way to the desired
location and (b) whose legal title is held by the entity; i.e., the entity bears the
risk of loss on inventory in transit. The following are the most common shipping
terms:
a) FOB shipping point - Legal title and risk of loss pass to the buyer when
the seller delivers the goods to the carrier. The buyer must include the
goods in inventory during shipping.
b) FOB destination - Legal title and risk of loss pass to the buyer when the
seller delivers the goods to a specified destination. The seller must
include the goods in inventory during shipping.
2) Goods out on consignment - A consignment sale is an arrangement between
the owner of goods (consignor) and the sales agent (consignee). Consigned
goods are not sold but rather transferred to an agent for possible sale. The
consignor records sales only when the goods are sold to third parties by the
consignee.
a) Goods out on consignment are included in the consignor’s inventory at
cost. Costs of transporting the goods to the consignee are inventoriable
costs, not selling expenses.
b) The consignee never records the consigned goods as an asset.
54 SU 2: Measurement, Valuation, and Disclosure: investments and Short-Term Items
Entity A's December 31, Year 1, warehouse inventory physical count results in the amount of inventory of $50,000. The '...
following is additional information regarding Entity A’s year-end inventory:
During'the year, Entity A consigned goods with a total cost of $60,000 to Entity B (the consignee). The annual
statement that was sent from Entity B to Entity A states that 60% of the consignment goods woro sold for.
$42,000. ..■ =
® Merchandise costing $40,000 shipped FOB shipping point from a vendor on December 29, Year 1, and was
received by Entity A on January 4, Year 2.
. ® Merchandise costing $70,000 shipped FOB destination from a vendor on December 30, Year 1, and was
received by Entity A on January 5, Year 2.
.© The goods billed to the. customer FOB destination on December 27, Year 1, had a cost of $25,000; The goods
. were shipped by Entity A on December 28, Year 1, and were received by the customer on January 3, Year 2.
In the December 31, Year 1, balance sheet, Entity A reports an inventory amount of $1.39,000. This amount consists of
. Warehouse physical inventory count $ 50,000.
Goods held on consignment ($60,000 * 40%) 24,000
Merchandise shipped FOB shipping point (title and risk of loss passed to Entity A on December 29,
. Year 1, at the time of shipment) • : 40,000
Goods shipped FOB destination to customer (title and the risk of loss will pass to the customer only on
January 3, Year 2) • S .. . .. . . 25,000
December 31, Year 1, Inventory balance $139,000
5. Inventory Estimation
a. An estimate of inventory may be needed when an exact count is not feasible, e.g., for
interim reporting purposes or when inventory records have been destroyed. The
gross profit method may be used for inventory estimation.
1) Gross profit margin (gross profit percentage) equals gross profit divided by
sales.
Gross profit margin (%) = Gross profit Sales
KV--V " -i EXAMPLE ''
A retailer needs to estimate ending inventory for quarterly reporting purposes. The firm's best estimate of the gross
percentage is its historical rate of 25%. The followihg additional information is available: . . . .
Net sales $1,000,000
: Purchases 300,000
Beginning inventory. 800,000
Beginning inventory . ; $ 800,000
Purchases .;r- =; 300,000
. .. Goods available for sale $1,100,000
. ; Sales'. • ■ ' .v $1,000,000
Gross profit (Sales * Gross profit margin) (250,000)
Cost of goods sold [Sales * (1 - Gross profit margin)] . (750,000) :
Ending inventory . v : $ 350,000
6. Inventory Errors
a. These errors may have a material effect on current assets, working capital (current
assets minus current liabilities), cost of goods sold, net income, and equity. A
common error is inappropriate timing of the recognition of transactions.
1) If a purchase on account is not recorded and the goods are not included in
ending inventory, cost of goods sold (Bl plus purchases minus El) and net
income are unaffected. But current assets and current liabilities are
understated.
SU 2: Measurement, Valuation, and Disclosure: Investments and Short-Term Items 55
2) If purchases and beginning inventory are properly recorded but items are
excluded from ending inventory, cost of goods sold is overstated. Net income,
inventory, retained earnings, working capital, and the current ratio are
understated.
b. Errors arising from recording transactions in the wrong period may reverse in the
subsequent period.
1) if ending inventory is overstated, the overstatement of net income will be offset
by the understatement in the following year that results from the overstatement
of beginning inventory.
c. An overstatement error in year-end inventory of the current year affects the
financial statements of 2 different years.
1) The first year’s effects may be depicted as follows:
Balance Sheet Effects Income Statement Effects
Current assets:
Ending inventory
Equity:
Retained earnings, 12/31/Yr 0
Plus: Net income
Retained earnings, 12/31/Yr 1
Net sales revenue
Beginning inventory
Plus: Cost of goods purchased/produced
Goods available for sale
$XXX,XXX
$XXX,XXX
XXX,XXX
XXX,XXX
Overstated f
$XXX,XXX
Overstated t <—.......
Overstated t
Cost of goods sold
Gross profit
Expenses
Net income <----------
| Overstated!
Understated f
Overstated t
(XX,XXX)
Overstated |
Figure 2-1
2) At the end of the second year, retained earnings is correctly stated as follows:
Balance Sheet Effects Income Statement Effects
Overstated T
Current assets:
Ending inventory $XXX,XXX -
Equity:
Retained earnings, 12/31/Yr 1
Plus: Net income
Retained earnings, 12/31/Yr 2 $XXX,XXX
Overstated |
Understated I
Net sales revenue
Beginning inventory
Plus: Cost of goods purchased/produced XXX,XXX
Goods available for sale j Overstated!7
-> Minus: Ending inventory----------------- > $(XXX,XXX)
Cost of goods sold
> Gross profit
Expenses
- Net income <------------------------------------------------
$XXX,XXX
Overstated T
Understated i
(XX,XXX)
Understated j
Figure 2-2
Stop and review! You have completed the outline for this subunit. Study multiple-choice
questions 6 through 9 beginning on page 71.
2.3 INVENTORY - COST FLOW METHODS
1. Specific Identification Method
a. Specific identification requires determining which specific items are sold and
therefore reflects the actual physical flow of goods. This system is appropriate for
(1) items that are not ordinarily interchangeable and (2) items that are segregated for
a specific project. It can be used for blocks of investment securities or special
inventory items, such as automobiles or heavy equipment.
1) Specific identification is the most accurate method because it identifies each
item of inventory.
2) However, it requires detailed records and may not be feasible or cost effective.
56 SU 2: Measurement, Valuation, and Disclosure: investments and Short-Term Items
b. When the inventory items purchased or produced are identical and interchangeable,
specific identification is not appropriate. In such circumstances, several assumptions
about the flow of cost, such as average, FIFO, or LIFO, may be appropriate for the
measurement of periodic income. The method selected should be the one that, under
the circumstances, most clearly reflects periodic income.
2. Average Method
a. The average method assumes that goods are indistinguishable and are therefore
measured at an average of the costs incurred. The average may be calculated on the
periodic basis or as each additional purchase occurs (perpetual basis).
1) The moving-average method is used under the perpetual inventory accounting
system. It requires determination of a new weighted-average inventory cost
after each purchase. This cost is used for every sale until the next purchase.
EXAMPLE
The following data relate to Entity A’s Year. 1 activities:
Number Purchase price Sale price
Pate:. Transaction .-. of units per unit ($) .. per unit ($)
Beginning balance 100 20
Purchase 20 32
Sale 70 . ■ ' 40
■ Purchase ■ 30 14 V;
-. Sale 40 24
Under the moving-average method, the year-end inventory and Year 1 cost of goods sold are calculated as follows: . ■
Date Activity Units Price
: Cost of inventory .■ r
purchased/sold i Inventory total balance
On-hand
units Cost per unit .
January 1 Beg. bal. 100 $20 $2,000 (100 x 20) 100 $20
March 1 Purchase 20 . . $32 $640 - 20 x $32 . $2,640 (2,000 + 640) 120 $22 ($2,640 + 1.20)
April 1 Sale 70 $22 ($1,540) = 70 x $22 . $1,100 (2,640-1,540) 50 $22 ($1,100 + 50);
June 1. Purchase 30. $14 . . $420 - 30 x $14 $1,520(1,100 + 420) 80 . $19 ($1,520+ 80)
October 1 Sale 40 $19 ($760) = 40.x $19. $760 (1,520-760) 40 • $19 ($760 + 40)
The cost of inventory on December 31, Year 1, is $760. The Year 1 cost of goods sold is $2,300.
Beginning inventory $2,000
Purchases ($640+. $420). . . 1,060
Ending inventory (760)
Cost of goods sold ($1,540+ $760) . $2,300
2) The weighted-average method is used under the periodic inventory
accounting system. The average cost is determined only at the end of the
period. The weighted-average cost per unit is used to determine the ending
inventory and the cost of goods sold for the period. It is calculated as follows:
Cost of beginning inventory ($) + Cost of purchases during the period ($)
Units in beginning inventory + Number of units purchased during the period
' EXAMPLE
Under the weighted-average method, Entity A’s ending inventory and Year 1 cost of goods sold are determined as
■follows:.-; •
First, the weighted-average cost per unit is calculated.
Cost of beginning inventory + Cost of purchases during the period ^
; Units in beginning inventory + Number of units purchased :
$2,000 + $1,060
' 100 + 20 + 30 : : *
- Continued on next page -
January. 1
March 1
April 1
June 1
October 1
SU 2: Measurement, Valuation, and Disclosure: Investments and Short-Term Items 57
EXAMPLE - continued
Second, the ending inventory and Year 1 cost of goods sold are calculated using the weighted-average cost per unit
(WACPU).
. . . Beginning inventory. •: ■ $2,000. ' ' ■. -
Purchases 1,060 ■ .. ; -
Ending inventory (816) (40 x $20.40) = (WACPU x Units in ending inventory) . . .
Cost of goods sold $2,244 (110 x $20.40) - (WACPU x Units sold during the period)
3. First-in, First-out (FIFO)
a. This method assumes that the first goods purchased are the first sold. Thus, ending
inventory consists of the latest purchases.
b. Cost of goods sold includes the earliest goods purchased.
c. Under the FIFO method, year-end inventory and cost of goods sold for the period are
the same regardless of whether the perpetual or the periodic inventory accounting
system is used.
EXAMPLE
The number of units in Entity A’s ending inventory is 40. Under the FIFO method, the cost of these units is the cost of the
latest purchases ($740).
Price Total
Date of purchase Units per unit cost
June 1, Year 1 30 $14 $420
March 1, Year 1 10 $32 $320
Ending inventory 40 $740
The Year 1 cost of goods sold is $2,320.
Beginning inventory $2,000
Purchases ($640 + $420) 1,060
Ending inventory (740)
Cost of goods sold $2,320
NOTE: The results are the same under the periodic and perpetual systems.
4. Last-in, First-out (UFO)
a. The LIFO (iast-in, first-out) method assumes the newest items of inventory are sold
first. Thus, the items remaining in inventory are the oldest. Under the LIFO method,
the perpetual and the periodic inventory accounting systems may result in different
values for year-end inventory and cost of goods sold.
1) Under the periodic inventory accounting system, the calculation of inventory and
cost of goods sold are made at the end of the period.
v; EXAMPLE Y.
The number of units in Entity A’s ending inventory is 40. Under the LIFO method, the cost of those units is the cost of the
earliest: purchases (beginning inventory) of $800 (40 units * $20). The Year 1 cost of goods sold is $2,260:%:
Beginning inventory $2,000
Purchases ($640 + $420) •• 1,060
Ending inventory (800)
"Cost of goods sold $2,260
58 SU 2: Measurement, Valuation, and Disclosure: Investments and Short-Term Items
2) Under the perpetual inventory accounting system, cost of goods sold is
calculated every time a sale occurs and consists of the most recent (latest)
purchases.
EXAMPLE
Date Activity Units
Cost
per unit
Cost of inventory
purchased/sold Inventory total balance
Number
of units
January 1 Beg. bal. 100- $20 - 100 x $20 = $2,000 100
March 1 Purchase $32 20 x $32 == $640
January 1, layer 100 x $20 = $2,000
March 1, layer 20 x $32 = 640
$2,640
120
April 1 Sale k7oM
20 x $32 =■ $ 640
50 x $20 =. . 1,000
$(1,640)
January 1, layer 50 * $20 ~ $1,000 50
June 1 Purchase 30 $14 .30 x.$14 = $420 .
January 1, layer . 50 x $20 = $1,000
June 1, layer 30 x $14 = ■■ 420
$1,420
80
October 1 Sale 40
30 * $14 = $ 420
10 x $20 -200.
$(620)
January 1, layer 40 x $20 - $800 40
Entity A's cost of ending inventory is $800, and the Year 1 cost of goods sold is $2,260 ($1,640 + $620).
NOTE: The results of the.LIFO method under the perpetual and periodic systems are the same in this example but may.
differ in other situations.
IFRS Difference
LIFO is not permitted.
©
5. Cost Flow Methods - Comparison
a. The cost flow mode! selected should be the one that most clearly reflects periodic
income.
EXAMPLE
Follows are Entity A's varying results under each of tho five cost flow methods:
■ Ending Cost of
Inventory Goods Sold
Moving average $760 ;■■■:? $2,300.
•; Weighted average - 816 2,244
FIFO 740 2,320 V -:
■v-..- LIFO periodic 800 2,260. ■-
■ ; LIFO perpetual . 800 :v--2,260 ^
b. An advantage of FIFO is that ending inventory approximates current replacement cost.
1) A disadvantage is that current revenues are matched with older costs.
c. Under LIFO, management can affect net income with an end-of-period purchase that
immediately alters cost of goods sold.
1) A last-minute FIFO purchase included in the ending inventory has no such effect.
SU 2: Measurement, Valuation, and Disclosure: Investments and Short-Term Items 59
d. Under LIFO, if fewer units are purchased than sold,
1) The beginning inventory is partially or fully liquidated and
2) Old costs are matched against current revenues in the year’s income statement.
e. In a time of rising prices (inflation), use of the LIFO method results in the lowest
year-end inventory, the highest cost of goods sold, and the lowest gross profit. LIFO
• assumes that the oldest (and therefore the lowest-priced) goods purchased are in
year-end inventory, and that cost of goods sold consists of the latest (and therefore
the highest-priced) goods purchased.
1) The results for the FIFO method are the opposite of those for the LIFO method.
Q leim
0,5 TIPs During a period Gross Profit
of inflation Ending inventory Cost of Goods Sold (Net Income)
LIFO Lowest Highest Lowest
K.. FIFO Highest Lowest Highest
Stop and review! You have completed the outline for this subunit. Study multiple-choice
questions 10 through 13 beginning on page 73.
2.4 INVENTORY MEASUREMENT IN THE FINANCIAL STATEMENTS - LOWER OF COST OR
MARKET
1. Statement of Rule
a. In the annual financial statements, inventory is measured at the lower of cost or
market (LCM). Inventory must be written down to market if its utility is less than its
cost at the end of the annual reporting period.
1) The difference (write-down) should be recognized as a loss in a separate line
item of cost of goods sold in the current-period income statement.
2) Reversals of write-downs of inventory are prohibited in subsequent periods.
2. Market
a. Market is the current cost to replace inventory, subject to certain limitations. Market
should not (1) exceed a ceiling equal to net realizable value (NRV) or (2) be less
than a floor equal to NRV reduced by an allowance for an approximately normal
profit margin.
b. Net realizable value is the estimated selling price in the ordinary course of business
minus reasonably predictable costs of completion and disposal.
1) Thus, current replacement cost must not be greater than NRV or less than NRV
minus a normal profit.
EXAMPLE of How to Calculate Inventories, Market, and the Year-Erid Measurement
The following information is related to a company’s year-end inventories: .
Cost per inventory unit Item A v Item B . Item C
Estimated selling prico $80. $7° $44
Minus: Cost of completion (20) -■’v V .(3)-;^: ■ ^'=j-
Minus: Cost of disposal J 6 ) J 5 ) J 2 )
NRV (ceiling) $54 $65 $39
Minus: Norma! profit margin J 3 ) _(7) _(4)
NRV-NPM (floor) 51 $58 $35
Current replacement cost (CRC) $53 $55 $40
(a) Market $53 Ceiling > CRC > Floor $58 Floor > CRC' $39 CRC > Ceiling
(b) Historical cost per unit $50 $60 ' $45
Lower of cost (b) or market (a) Cost < Market M Market < Cost J H Market < Cost
60 SU 2: Measurement, Valuation, and Disclosure: Investments and Short-Term Items
3. Applying LCM
a. Depending on the nature of the inventory, the LCM rule may be applied either directly
to each item or to the total of the inventory. The method should be the one that most
clearly reflects periodic income.
1) Once inventory is written down, the reduced amount is the new cost basis.
2) Most entities use LCM by item. This method is required for tax purposes.
^ <V;' ^ example-;- :-y :-=y
. Allotrope Co.: has the following information about its inventory at the end of the fiscal year:
Historical cost . $100,000
Current replacement cost 82,000
Net realizable value (NRV) . 90,000
Normal profit margin 5,000
Inventory is measured at tho lower of cost or market (current replacement cost subject to certain limitations). Market cannot
be higher than NRV ($90,000) or lower than NRV reduced by a normal profit margin ($90,000 - $5,000 - $85,000). Thus,
market is $85,000. (The current replacement cost of $82,000 is below the floor.) Because market is. lower than cost, the
inventory is reported in the balance sheet at market of $85,000. The write-down of inventory of $15,000 ($100,000 -
$85,000) is recognized as a loss in the income statement. The journal entry is as follows:
Loss from inventory write-down . $15,000
Inventory $15,000
IFRS Difference
Inventories are measured at the lower of cost or net realizable value (NRV).
NRV is the estimated selling price less the estimated costs of completion and
disposal. NRV is assessed each period. Accordingly, a write-down may be
reversed but not above original cost The write-down and reversal are
recognized in profit or loss.
Using the data from the example above, NRV ($90,000) is lower than cost
($100,000). Thus, the inventory is reported in the statement of financial
position at its NRV ($90,000). The write-down of inventory of $10,000
($100,000 - $90,000) is recognized in profit or loss. The journal entry is as
follows:
Loss from inventory write-down $10,000
Inventory $10,00

No comments:

Post a Comment